Sie sind auf Seite 1von 65

PEDIATRICS

1 A newborn FT was noted to be pale at 4th month of life. Iron deficiency is unlikely in this condition because
infants have sufficient stores to meet their iron requirement for:
A. 2-3 months
B. 4-6 months
C. 7-8 months
D. 10-12 months
(Problem Solving) - GIT
B – It is by 6 months that iron should be supplemented among healthy full terms.
(Nelson’s Textbook of Pediatrics, 17th ed, 2004, p. 156)

2 The breastfed baby of a pure vegetarian mother may develop:


A. Xerophthalmia
B. Diarrhea & Dementia
C. Osteomalacia
D. Anemia
(Problem Solving) - GIT
D - Strict Vegan diets contain no eggs, meat or milk products making this deficient in Vitamin B
12.
(Chap. 446 p. 1612)
Nursing Vegan mothers must be given B12 to prevent. Methylmalonic academia and anemia
in their infants. (166)
(A) Xerophthalmia is Vitamin A deficiency and for which Vitamin A rich sources are the
vegetables.
(B) Diarrhea and dementia are signs of niacin deficiency (Pellagra) B vitamins come from
grains and vegetables.
(C) Osteomalacia is Vitamin D deficiency (Rickets)
(Nelson’s Textbook of Pediatrics, 17th ed, 2004, Chap. 446 Table 44-I)

3 By 6th month of age micronutrients must be started. Foremost among these is that nutrient that
prevents:
A. Xerophthalmia
B. Scurvy
C. Anemia
D. Goiter
(Problem Solving) - GIT
C - All nutrient needs of infants must be met by 6 months. But during this time breast milk
volume and iron stores may not be adequate to accommodate the demands of growth – Iron deficiency
anemia may ensue.
(Nelson’s Textbook of Pediatrics, 17th ed, 2004, p. 164)

4. Chronic intake of carotenoids may result in:


A. Pseudotumor cerebri
B. Yellow skin and sclerae
C. Yellow skin
D. Cranial nerve palsy
(Recall) - GIT
C - Carotene (yellow pigment); carotenemia although non-toxic is due to deposition of carotene
pigments in the skin but not the sclerae,
(B) Icteresia and jaundice is yellowish discoloration of the sclerae and skin secondary to
deposition of bilirubin due to a pathology in Bilirubin metabolism hemolytic of hepatobiliary disease
(A and D) Pseudotumor cerebri and cranial nerve palsy are CNS manifestations of Vitamin A
toxicity after chronic intake of >100,000/u/day vitamin A.
(Nelson’s Textbook of Pediatrics, 17th ed, 2004, p. 181)

5. A 3-hour old newborn with a prenatal history of maternal hydramnios was noted to have frothing of
mouth and nose with circumoral cyanosis. You anticipate that:
A. There is inability to pass the nasogastric tube
B. Presence of scaphoid abdomen

1
C. Referral to ENT will be done
D. All of the above
(Problem Solving) - GIT
A - In early onset respiratory distress, inability to pass an NGT suggests esophageal atresia with
TEF. This is a surgical problem not seen by ENT
(C) maternal polyhydramnios is more associated with TEF rather than diaphragmatic hernia
(B)
(Nelson’s Textbook of Pediatrics, 17th ed, 2004,Chapter 300)

6. Which of the micronutrients does not have recognized anti-infective properties?


A. Vitamin A
B. Vitamin D
C. Iron
D. Zinc
(Recall) - GIT
B - Deficiency of any essential nutrient may result in failure to thrive and accompanying lack of
immune protection. However, infections are more common in children with Vitamin. A, Iron, Zinc
deficiencies. These 3 have roles in the immune system.
(Nelson’s Textbook of Pediatrics, 17th ed, 2004, Chapter 44)

7. A 1-1/2 year old is discovered to have a bottle of alkali solution in his mouth. The bottle was noted to
be half empty. No external signs on the child's face were seen. Your advice is to bring the child to the
ER. There must be prior administration of:
A. Emetic
B. Antiemetic
C. Milk
D. Laxative
(Problem Solving) - GIT
C - Milk calms the child and dilutes the alkali
(A) don’t induce emesis
(Nelson’s Textbook of Pediatrics, 17th ed, 2004, (Chapter 308.2)

8. A 3 year old accidentally ingested a coin. A chest x-ray was taken. In contrast to foreign body
trachea, the coin in the esophagus as seen on radiograph will show:
A. Edge of the coin in AP view
B. Edge of coin on lateral view
C. Flat surface in AP view
D. A and C
(Problem Solving) - GIT
D - (Nelson’s Textbook of Pediatrics, 17th ed, 2004, Chapter 308.1)

9. The WHO recommends the use of ORS in developing countries to have a sodium concentration of
_____ mmol/L:
A. 90
B. 100
C. 110
D. 120
(Recall) - GIT
A - 90 mmol/l
Above 90 is hyperosmolar
(Nelson’s Textbook of Pediatrics, 17th ed, 2004, p. 250)

10. A 3-week old with essentially normal birth history had episodes of intermittent vomiting after feeding.
If pyloric stenosis is being considered, you expect the following EXCEPT:
A. Hypochloremic alkalosis
B. Bilous vomiting
C. Gastric peristaltic wave
D. Olive-shaped RUQ mass in abdominal palpation
(Problem Solving) - GIT
C - The hallmark of gastric obstruction is non-bilious vomiting.
(Nelson’s Textbook of Pediatrics, 17th ed, 2004, Chapter 310)

11. A one day old had bilous vomiting. He was noted to be slightly jaundiced. The abdomen was not
distended but there was occasional visible peristaltic nerves on the abdominal wall. Plain abdomen x-ray
showed double-bubble sign. The obstruction is on what level?
A. Distal esophagus
B. Gastric
C. Duodenal

2
D. Colonic
(Problem Solving) - GIT
C - (Nelson’s Textbook of Pediatrics, 17th ed, 2004, p. 1233)
When the obstruction is in the duodenum beyond the Ampula of Vater – vomitus is bilous.
The Ampulla of Vater is the site where bile exits.

12. A 2-year old with head trauma underwent a neurosurgical procedure. At the PICU he had massive
hematemesis. You would consider:
A. Curling’s ulcer
B. H. pylori infection
C. Cushing’s ulcer
D. B and C
(Problem Solving) - GIT
C - (Nelson’s Textbook of Pediatrics, 17th ed, 2004, Chapter 316.1)
Gastric hypersecretion is associated with head trauma and severe CNS disorders
(A) Curling’s ulcers are associated with severe burns
(B) The course is too acute for H. pylori infection

13. An 11 year old Tanner stage 2 female developed epigastric pain / 8 hours later there was fever
nausea and vomiting. She passed 2 soft bowel movements. In the clinic, she limps and abdominal
palpation, there was generalized guarding. Most likely, she has:
A. Pelvic inflammatory disease
B. Ruptured ectopic pregnancy
C. Appendicitis
D. Mesenteric adenitis
(Problem Solving) - GIT
C - All choices are differentials of appendicitis
(A) PID presents with vaginal discharge
(B) Pregnancy is unlikely for Tanner 2
(D) Mesenteric adenitis follows a week of respiratory infection
(Nelson’s Textbook of Pediatrics, 17th ed, 2004, Chapter 324)

14. A 2 year old previously well child had intermittent crying episodes and projectile vomiting 12 hours
ago. There was gassy abdominal distention and passage of maroon-colored stools. You would:
A. Give antiamebics and antiemetics
B. Do abdominal x-ray and refer to surgery
C. Give antibiotics
D. All of the above
(Problem Solving) - GIT
B - The diagnosis is intussusception
(A) and (C) are not employed in intussusception
(Nelson Textbook of Pediatrics, 17th ed, 2004)

15. The GI malignancy prevented by immunization is:


A. Gastric carcinoma
B. Colonic carcinoma
C. Hepatic carcinoma
D. Pancreatic carcinoma
(Recall) - GIT
C - Hepatitis B directly increases the risk of Hepatocarcinoma in later life. This is prevented by
vaccination. There are no known vaccines for preventing the onset of the other cancers. Gastric cancer
from H. pylori gastritis is well documented. So far there are no H. pylori vaccines.
(Nelson’s Textbook of Pediatrics, 17th ed, 2004, p. 1328)

16. Prolonged antibiotic therapy can result to bleeding with the following laboratory results:
A. Normal PT, normal PTT
B. Prolonged PT, prolonged PTT
C. Prolonged PT, normal PTT
D. Normal PT, prolonged PTT
(Problem Solving) – Hema/Onco
C - Prolonged antibiotic therapy can lead to gut sterilization leading to reduced synthesis of
Vitamin K – dependent clotting factors (Factors II, VII, IX & X, protein C and protein S). This reduction of
clotting factors of the extrinsic limb of coagulation will lead to prolongation of Prothrombin Time with
normal PTT.
(Nelson’s Textbook of Pediatrics, 17th ed, 2004)
17. The CBC of a 7-yer old male with epistaxis and ecchymoses revealed Hgb 67 g/dl, Hct 18%, WBC
50,000, Neutrophils 5%, Lymphoblast 95%, Platelet Count 20,000. What is you primary consideration?
A. Aplastic anemia

3
B. Acute lymphocytic leukemia
C. Disseminated intravascular coagulation
D. Idiopathic thrombocytopenic purpura
(Problem Solving) – Hema/Onco
B - The anemia and thrombocytopenia are due to decreased production of erythroid and
megakaryocytic precursors resulting from blastic proliferation in the bone marrow. Aplastic
anemia (Choice a) is associated with pancytopenia. DIC (Choice C) doesn’t produce leukocytosis and
blasts in the peripheral smear. It is associated with the coagulation mechanism, not the hematopoietic
cells. ITP (Choice D) is only associated with thrombocytopenia. No leukocytosis and blasts are seen in
the peripheral smear.
(Nelson’s Textbook of Pediatrics, 17th ed, 2004)

18. Recurrent gum bleeding was noted in a 7-year old female. CBC and platelet count are normal,
Prothrombin time is normal but bleeding time and partial thromboplastin time are prolonged. The most
likely diagnosis is:
A. ITP
B. Hypoprothrombinemia
C. TTP
D. Von Willebrand Disease
(Problem Solving) – Hema/Onco
D - Von Willebrand disease is a disorder associated with mucocutaneous hemorrhages. The
disorder is due to deficiency of Von Willebrand factor, a glycoprotein that is synthesized in
megakaryocytes and endothelial cells. During normal hemostasis VWF adheres to the endothelial matrix
after vascular damage. Changes in the conformation of VWF cause platelets to be an adhere to VWF
resulting to platelet activation and recruitment of additional platelets. VWF also serves as the
carrier protein for plasma factor VIII. Severe deficiency of VWF can cause prolongation of bleeding time
and PTT. ITP (Choice A) results only to prolonged BT because the coagulation phase is not affected.
Hypoprothrombinemia (Choice B) results to decreased synthesis of Vitamin K – dependent factor causing
prolonged PT. ITP (Choice C) is a form of microangiopathic hemolytic anemia with thrombocytopenia.
(Nelson’s Textbook of Pediatrics, 17th ed, 2004, Chap. 469)

19. A 10-kg child with iron deficiency anemia should receive:


A. 20 mg elemental iron
B. 30 mg elemental iron
C. 60 mg elemental iron
D. 70 mg elemental iron
(Recall) – Hema/Onco
C - The therapeutic dose of elemental iron is 6 mkd.
(Nelson’s Textbook of Pediatrics, 17th ed, 2004, Chap. 447)

20. The following statement is true regarding brain tumors in childhood:


A. Hereditary syndromes are associated with increased incidence of brain tumors in 25%
of cases
B. Cranial exposure to ionizing radiation is associated with increased incidence of brain
tumor
C. Supratentorial tumors predominate among children aged 1-10 years
D. In general, there is a slight predominance of supratentorial tumor location in children
(Recall) – Hema/Onco
B - Cranial exposure to ionizing radiation has been shown to be associated with increased
incidence of brain tumors. This has been observed in pediatric acute lymphocytic leukemia who
underwent craniospinal prophylaxis. Hereditary syndrome (Choice A) are associated with increased
incidence only in 5% of cases. Supratentorial tumors predominate during the 1 st year of life thus Choice
C is incorrect. Generally, infratentorial tumors predominate slightly over supratentorial tumors making
Choice D incorrect.
(Nelson’s Textbook of Pediatrics, 17th ed, 2004, Chap. 489)

21. Neuroblastoma is a condition characterized by the following:


A. Malignancy most frequently diagnosed in infancy
B. Mixed embryonal neoplasm composed of three elements: blastoma, epithelia and
stroma
C. Classically presents with leukocoria
D. Diagnosis does not require a biopsy but is established characteristical clinical findings
(Problem Solving) – Hema/Onco
A - Neuroblastoma is an embryonal cancer of the peripheral sympathetic nervous system. It is
the third most common pediatric cancer accounting for about 8% of pediatric cases. It is the most
common malignancy in infancy accounting for 28-39% of neonatal malignancies. Mixed embryonal
neoplasm composing of three elements (Choice B) pertains to Wilm’s tumor. Leukoria (Choice C) is the

4
characteristic clinical presentation of retinoblastoma. Diagnosis does not require a biopsy (Choice D) in
retinoblastoma since characteristic ophthalmologic findings are sufficient. (Nelson’s Textbook of
Pediatrics, 17th ed, 2004, Chap. 490)
22. Among the following tumors, the one with the best over-all survival rate is:
A. Wilm’s tumor
B. Non-Hodgkin’s lymphoma
C. Hepatoblastoma
D. Neuroblastoma
(Problem Solving) – Hema/Onco
A - Prognosis of neuroblastoma is generally good. Survival in low risk group is 91-100%;
average group 75-98% Stage 4S carries 100% survival with supportive care only because the tumor
regresses spontaneously. Wilm’s tumor (Choice A) prognostic factors are tumor size, stage and
histology. More than 60% of patients with all stages generally survive. Non-Hodgkin’s lymphoma (Choice
B) is considered disseminated disease from the time of diagnosis. Hepatoblastoma (Choice C) if
unresected carries survival rate of 60%.
(Nelson’s Textbook of Pediatrics, 17th ed, 2004, Chap. 491)

23. Which of the following patterns noted on continuous monitoring of fetal heart rate is most indicative of
fetal distress?
A. Baseline variability with periodic acceleration
B. Increasing baseline variability
C. Early deceleration without baseline variability
D. Late deceleration without baseline variability
(Problem Solving) - Neonatology
D - Baseline variability with or without periodic acceleration of the heart rate is a sign of fetal well-
being. Increasing baseline variability may represent early compromise of fetal oxygenation. The early
deceleration pattern is due to pressure of the anterior fontanelle on the cervix and is not a sign of fetal
distress. The variable deceleration pattern indicates umbilical cord compression. The late deceleration
pattern signifies fetal hypoxemia.
(Behrman, ed. 13, p. 368)

24. A healthy premature infant who weighs 950 g (2 lb, 1 1/2 oz) is fed undiluted breast milk to provide
120 cal/kg per day. Over ensuing weeks the baby is most apt to develop:
A. Hypernatremia
B. Hypocalcemia
C. Blood in the stool
D. Metabolic acidosis
(Problem Solving) - Neonatology
B - Breast milk has much less calcium and phosphorus than do commercial formulas.
(Behrman, ed. 113, pp. 162-163)

25. An infant weighing 1400 g (3 lb) is born at 32 weeks gestation in a delivery room that has an ambient
temperature of 24'C. Within a few minutes of birth, this infant is likely to exhibit all the following EXCEPT:
A. Pallor
B. Shivering
C. A fall in body temperature
D. Increased respiratory rate
(Problem Solving) - Neonatology
B - A room temperature of 24’C provides a cold environment for preterm infants weighing less
than 1500 g. Aside from the fact that these infants emerge from a warm intrauterine environment. In
order to bring body temperature back to normal they must increase their metabolic rate; ventilation in
turn, must increase proportionally to ensure adequate oxygen supply. Infants rarely shiver in response to
a need to increase heat production.
(Behrman, ed. 113, p. 363)

26. Initial examination of a full tem infant weighing less than 2500 g (5 lb, 8 oz) shows edema over the
dorsum of her hands and feet. Which of the following findings would support a diagnosis of Turner's
syndrome?
A. A liver palpable to 2 cm below the costal margin
B. Tremulous movements and ankle clonus
C. Redundant skin folds at the nape of the neck
D. A transient, longitudinal division of the body into a red half and a pale half
(Problem Solving) - Neonatology
C - Turner’s syndrome is a genetic disorder with the 45XO karyotype being most common. At
birth affected infants have low weights, short stature, edema over the dorsum of hands and feet and
loose skin folds at the nape of the neck.
(Behrman, ed. 13, pp. 264-266. 1236-1237)

5
27. Object permanence is not present in a 2 months old, whose response to dropping a ball is:
A. Staring descending as the ball descends
B. Eyes descending as the ball hits the ground
C. Crying when the ball hits the ground
D. Smiling at the game of the hide-and-seek

(Problem Solving) - Neonatology


A - Out of sight out of mind is the characteristic response of a 2 month old. Object permanence
appears at approximately 8 months of age. This is also called object constancy.
(Nelson Textbook of Pediatrics, 15th ed, Chap. 11))

28. The ability to manipulate small objects with the pincer grasp is usually noted at what age?
A. 0 to 2 months
B. 3 to 5 months
C. 6 to 7 months
D. 8 to 9 months
(Recall) - Neonatology
D - The pincer grasp, which is noted at age 8 to 9 months, along with increasing mobility,
enables an infant to explore the environment.
(Nelson Textbook of Pediatrics, 17th ed, 2004)

29. A developmentally normal child who is able to run, build a tower of two cubes, pretend play with a
doll and speak in two-word sentences is what age?
A. 19 months
B. 15 months
C. 14 months
D. 24 months
(Problem Solving) - Neonatology
A - (See Table 11-3, Chapter 11, Nelson Textbook of Pediatrics, 15 th ed)

30. A developmentally normal child who is just able to sit without support, transferobjects from hand to
hand, and speak in a monosyllabic babble is probably what age?
A. 2 months
B. 4 months
C. 9 months
D. 6 months
(Problem Solving) - Neonatology
D - (Nelson Textbook of Pediatrics, 15th ed, See Table 11-3, Chap 11)

31. This primitive reflex is observed in a normal one year old:


A. Tonic neck reflex
B. Parachute reflex
C. Palmar grasp
D. Placing reflex
(Recall) - Neurology
B - Among these 4 choices, it is B that persists normally beyond the neonatal period. In fact, the
parachute reflex persists for life.
(Nelson’s Textbook of Pediatrics, 17th ed, 2004, p. 1978)

32. Which of the following case scenarios merit an EEG as an initial test as part of the
neurodiagnostic evaluation?
A. Febrile seizure
B. First non-febrile seizure
C. Meningitis
D. Intracranial SOL
(Problem Solving) - Neurology
B - While the first febrile seizure is generally a benign one, an EEG is requested if it recurs. The
EEG provides characterization of seizure types which allows for the specific medical or surgical
management. A lumbar puncture with CSF analysis would have confirmed meningitis. A brain CT scan
would have demonstrated the intracranial SOL.
(Nelson’s Textbook of Pediatrics, 17th ed. ; Behrman, Kliegman & Jenson, 2004, p. 1978)

33. A 2-year old boy was admitted because of low to moderate grade fever of 3 weeks, on and off frontal
headache of 1 week, squinting of 1 day, one episode of generalized seizure of 2 minute duration
6 hours prior to admission. No medical consult done. No medications given except paracetamol.
Which of the following clinical consideration is NOT COMPATIBLE with this history?
A. Acute meningococcal meningitis
B. TB meningitis

6
C. Cryptococcal meningitis
D. Brain abscess of otogenic origin
(Problem Solving) - Neurology
A - Except for A, all the rest are compatible of the history, presenting with the clinical
manifestations of at least 2 weeks.
(Nelson’s Textbook of Pediatrics, 17th ed., 2004, pp. 965 ; 2040-44)

34. A mother calls to inform you that her previously well 4-year old child has been complaining of
headaches for about a month. For the past two weeks he has been keeping his hand in a tilted position,
and for the past few days he has been vomiting in the morning. The most likely diagnosis is:
A. Meningitis
B. Degeneration brain disease
C. Brain abscess
D. Brain tumor
(Problem Solving) - Neurology
D - Frequently, meningitis or CNS infections will present with fever, headache, and signs of
irritability. Brain abscess, because it behaves like an intracranial SOL, will present as low grade fever,
headache, and localizing signs. The hallmark of neurodegenerative disease is progressive deterioration
of neurologic functions with loss of speech, vision, hearing, or locomotion, often associated with seizures,
feeding difficulties, and impairment of intellect. Generally, brain tumors present with signs and symptoms
relating to increased intracranial pressure (vomiting, lethargy, irritability) and focal neurologic deficits.
Within the 1st year of life, supratentorial tumors predominate and include, most commonly, choroids
plexus complex tumors and teratomas. From 1-10 years of age, infratentorial tumors predominate, owing
to the high incidence of juvenile pilocytic astrocytoma and medulloblastoma. After 10 years of age,
supratentorial tumors again predominate, with the diffuse astrocytomas, most common.
(Nelson’s Textbook of Pediatrics, 17th ed, 2004, pp. 1703, 2029, 2038, 2047)

35. Clinical evidence backs up the use of IV dexamethasone as an adjunctive therapy in acute
meningitis caused by _____:
A. Neisseria meningitidis
B. Streptococcus pneumoniae
C. Hemophilia influenza
D. Listeria monocytogenes
(Recall) - Neurology
C - Data support the use of IV dexamethasone, 0.15 mg/kg/dose given every 6 hours x 2 days
with bacterial meningitis caused by Hemophilus influenzae type b, but not with other bacterial causes, in
terms of less fever, lower CSF protein and lactate levels, and a reduction in permanent auditory nerve
damage, as manifested by sensoneural loss.
(Nelson’s Textbook of Pediatrics, 17th ed, 2004, p. 2043)

36. A 12-year old child is admitted because of the sudden onset of coma. The child had been well until
about 6 hours prior to admission, when he began to complain of a headache. The headache became
more severe, and the child lapsed into coma. Physical examination: T = 38.2'C, flaccid and comatose.
CSF: bloody: after centrifugation, the fluid appears xanthochromic, RBC = 3,000, WBC 7/mm3 , protein
400 mg/dl, glucose is 62 mg/dl. The most likely etiology of the coma is:
A. Intraventricular hemorrhage
B. Subarachnoid hemorrhage
C. Viral encephalitis
D. Subdural effusion
(Problem Solving) - Neurology
B - The event is something acute, dramatic, catastrophic so the choices would only be between A
and B. Intracranial bleeding may occur in the subarachnoid space or the bleeding may be primarily
located in the parenchyma of the brain. Subarachnoid bleeding characterized by severe headache,
nuchal rigidity, and progressive low of consciousness, and intracerebral bleeding is a common event in
premature infant (intraventricular hemorrhage). Rupture of an arteriovenous malformation (AV mal) may
occur at any age, and causes severe headache, vomiting, nuchal rigidity caused by subarachnoid
bleeding, progressive hemiparesis and a focal or generalized seizure.
(Nelson’s Textbook of Pediatrics, 17th ed, 2004, pp. 2036, 562)

37. The metaphyseal ends of long bones are common sites of osteomyelitis. This condition occurs
because:
A. Relative anoxia promotes bacterial growth
B. There is blood pooling and reduced phagocytic activity
C. They are closer to the skin surface
D. They are common sites of trauma
(Problem Solving) – Musculoskeletal Disorders

7
B -The unique anatomy and circulation of the ends of long bones results in the predilection for
localization of blood borne bacteria. In the metaphysic, nutrient arteries branch into non-anastomosing
capillaries under the physics, which make a sharp loop before entering venous sinusoids draining into the
marrow. Blood flow in this area is sluggish and provides an ideal environment for bacterial seeding.
(Nelson’s Textbook of Pediatrics, 17th ed., 2004, pp. 2297-2298)

38. It is the most common primary malignant bone tumor in children and adolescents, which shows a
"sunburst" pattern on radiographs:
A. Ewing sarcoma
B. Osteosarcoma
C. Osteochondroma
D. Osterblastoma
(Recall) – Musculoskeletal Disorders
B - Osteosarcoma is the most common primary malignant bone tumor in children and
adolescents, followed by Ewing sarcoma. In children younger than 10 years of age, Ewing sarcoma is
more common than osteosarcoma. Both tumor types occur most frequently in the 2 nd decade of life.
(Ref. Nelson’s Textbook of Pediatrics, 17th ed., 2004, pp. 1717)

39. An adolescent male basketball enthusiast consults you with a painful bump below his right knee. He
denies fever or trauma. Which of the following is the most likely diagnosis?
A. Legg-Calve Perthes Disease
B. Osteoid osteoma
C. Osgood-Schlatter disease
D. Osteomyelitis
(Problem Solving) – Musculoskeletal Disorders
C – Osgood-Schlatter disease occurs in active children, particularly during late childhood or
adolescence, especially in athletes, and consists of the tearing of cartilage from the tibial tuberosity by the
ligamentum patellae. The child presents with pain and swelling at the site of one or both tibial tubercles.
Rest, restriction of activities, and occasionally, a knee immobilizer may be necessary combined with
isometric exercise program. Complete resolution of symptoms through physiologic healing (physeal
closure) of the tibia tubercle usually requires 12-24 months.
(Nelson’s Textbook of Pediatrics, 17th ed., 2004, pp. 2272).

40. An overweight adolescent male complains of pain in the medial aspect of his knee. He denies
trauma, and he has not had a fever. The most likely diagnosis is:
A. Toxic synovitis
B. Legg-Calve-Perthes disease
C. Medial collateral ligament strain
D. Slipped capital femoral epiphysis
(Problem Solving) – Musculoskeletal Disorders
D - Slipped capital femoral epiphysis (SCFE) is the most common adolescent hip disorder with an
unknown cause, in which there is a displacement of the femoral head from the femoral neck prior to
epiphyseal closure. Common in obese adolescent boys, it presents with pain, limp, or refusal to walk.
The pain may be referred to the knee or thigh. Legg-Calve-Perthes Disease, avascular necrosis of the
femoral head presents with joint stiffness, hip and pain in the hip, thigh, knee, or groin of several weeks to
months. Boys between 1-12 years (average 7 years) are most commonly affected. Toxic synovitis is a
transient inflammatory arthritis of the hip associated with fever.
(Nelson’s Textbook of Pediatrics, 17th ed., 2004, pp. 2276-2279)

41. This statement is NOT true about infective endocarditis:


A. In 90% of cases, the causative agent is recovered from the first 2 blood cultures
B. Timing of phlebotomy is important because bacteremia occurs only during the febrile
state
C. Antimicrobial pretreatment of the patients reduces the yield of blood cultures to 50-
60%
D. Laboratory should be notified that endocarditis is suspected so that the blood can be
cultures on enriched media for more than 7 days
(Problem Solving) - Cardiovascular
B - Timing of collection is not important because bacteremia can be expected to be relatively
constant.
(A) It is true that in 90% of cases, the causative agent is recovered from the first 2 blood
collection
(C ) It is true that pretreatment with antimicrobials of the patients with bacterial endocarditis
reduces the yield of blood culture to 50-60%

8
(D) It is true that the laboratory should be notified that endocarditis is suspected so that if
necessary the blood can be cultured on enriched media for longer than 7 days to detect nutritionally
deficient and fastidious bacteria or fungi. And laboratory should be notified that the patient has received
antibiotics so that more sophisticated methods can be used to recover the offending organisms.
(Nelson’s Textbook of Pediatrics, 17th ed, 2004)

42. Painless small erythematous or hemorrhagic lesion on the palms and soles are classic lesion in:
A. Osler nodic
B. Janeway lesions
C. Roth spots
D. Spincter Hemorrhages

(Recall) - Cardiovascular
B - Janeway lesion are painless small erythematous or hemorrhagic lesions on the palms and
soles.
(A) Osler nodes are tender pea-sized intradermal nodule in the pads of the fingers and toes.
These lesions may represent vasculitis produced by circulating antigen antibody complexes
(C ) Sphincter hemorrhages are linear lesions beneath the nodes
(D) Roth spots – immune complex phenomena and seen in the eyes
(Nelson’s Textbook of Pediatrics, 17th ed, 2004)

43. Neonatal circulation is NOT characterized by:


A. In the presence of cardiopulmonary disease PDA may remain patent
B. Foramen ovale may persistently be functional
C. The wall thickens and muscle mass of the neonatal (L) and (R ) ventricles are almost
equal
D. The pulmonary vasculature is insensitive to changed pO4 and PC02 levels an
acidosis
(Problem Solving) - Cardiovascular
D - the pulmonary vasculature is very reactive to changes in pCO2, pO2 and pH by vigorous
vascular constriction
(A) in the presence of cardiopulmonary disease resulting to hypoxemia may cause the PDA to
remain open. Normal PDA functionally closes by the 10-15th hour of life
(B) Foramen ovale is functionally closed by the 3 rd months of life
(C ) the wall thickness and muscle mass of the ventricles right and left are almost equal.
Without the placenta, and the closure of the ductus venosus, the left ventricle is now coupled to the high
resistance systemic circulation whereas the right ventricle is now coupled with the low resistance
pulmonary circulation and the wall is slightly thickened as well
(Nelson’s Textbook of Pediatrics, 17th ed, 2004)

44. The clinical manifestation of large VSD in neonatal patients does not include:
A. Systolic murmur may not be audible
B. Dyspnea
C. Profuse perspiration
D. Recurrent pulmonary infection
(Problem Solving) - Cardiovascular
A - systolic murmur may not be audible this occurs only in small VSD this is due to the fact that
the left to right shunt may be minimal because of the higher right sided pressure
(B) Dyspnea happens because of excessive blood flow and pulmonary hypertension
(C ) profuse perspiration is a sign of heart failure secondary to high level of left ventricular
output heart rate and stroke volume are increased mediated by an increased level of sympathetic
nervous system stimulation and activity thus increasing the circulation of catecholamines combined with
increased work of breathing resulting in the elevation of in total body oxygen consumption often beyond
the oxygen transport ability of the circulation
(D recurrent respiratory infection secondary to the presence of “wet” lung syndrome that
serves as a niduos infection coupled with the disruption of the mucociliary clearance these will be
responsible for the recurrence of URTI
(Nelson’s Textbook of Pediatrics, 17th ed, 2004)

45. Which of the following cardiac anomaly is NOT present in Tetralogy of Fallot?
A. Pulmonary stenosis
B. ASD
C. Overriding of the aorta
D. Right ventricular hypertrophy
(Recall) - Cardiovascular

9
B - Atrial septal defect is NOT seen in patients with TOF. It is ventricular septal defect (VSD) is
the defect that is part of the defect and the VSD is frequently non restrictive and large frequently located
just below the aortic valve.
(A) Pulmonary stenosis leads to the obstruction of the Right ventricular outflow. The
pulmonary valve annulus may be of nearly normal size or may be quite small in size. The valve itself is
bicuspid and occasionally is the only site of the stenosis. In cases where the right ventricular outflow tract
is completely obstructed, pulmonary blood flow may be supplied by a patent ductus arteriosus (PDA) and
by major aortopulmonary collateral arteries arising from the aorta
(C ) Over riding of the aorta is part of the congenital defect
(D) Right ventricular hypertrophy is due to the degree of right ventricular outflow obstruction
(Nelson’s Textbook of Pediatrics, 17th ed, 2004)

46. A 3-year old boy was admitted to the ER because of difficulty of breathing. History revealed that he
developed high grade fever and sore throat 24 hours prior to consult with associated difficulty of
swallowing. Physical examination showed a very toxic looking boy, highly febrile, with labored breathing
and hyper extended neck and drooling of the saliva. The most plausible diagnosis of the above case is:
A. Acute infectious laryngitis
B. Acute epiglottitis
C. Acute laryngotracheobronchitis
D. Acute bacterial tracheitis

(Problem Solving) - Respiratory


B - Acute epiglotittis
This is a potentially lethal condition characteristically presenting with acute fulminating
course of high grade fever, sore throat, dyspnea and rapidly progressing respiratory obstruction. Drooling
of the saliva is frequently present and is due to difficulty of swallowing. Hyperextension of the neck is due
to his attempt to maintain the patency of the airway. This fatal disease is frequently caused by H.
influenzae.
(A) Acute infectious laryngitis is frequently caused by viral agents and the disease is usually
mild and non fatal. The onset of the disease is usually characterized by an upper respiratory tract
infection during which sore throat, cough and hoarseness appear. Respiratory distress is unusual except
in the very young infants where the airways are very compliant and small in caliber.
(C) Acute laryngotracheobronchitis “croup” is again frequently caused by viruses. Most of the
patients will present with upper respiratory tract infection with a combination of rhinorrhea,
pharyngitis, mild cough and low grade fever for 1-3 days before the appearance of the signs and
symptoms of upper airway obstruction. It starts with “barking” cough ,hoarseness and inspiratory stridor
which characteristically becoming worse at night and often recurring with decreasing intensity for several
days and completely resolves with in a week.
(D) Acute bacterial trachietis this entity is a form of bacterial infection of the upper airway and
does not involve the epiglottis. It is capable of causing life threatening airway obstruction. It is frequently
caused by staphylococcus aureus and other organisms like Moraxella catarrhalis, nontypable H.
Influenzae and anaerobic organisms have been implicated. It frequently occurs in children younger than 3
years of age.
A patient seen at the pediatric OPD clinic because of prolonged harsh “barky” cough that
lingered behind after a bout of viral infection not responsive to treatment i.e. bronchodilators and
mucolytics and disappears when the patient is asleep.
(Nelson’s Textbook of Pediatrics, 17th ed, 2004)

47. The mechanism of hypoxia in pulmonary edema is:


A. V/Q mismatch
B. Hypoventilation
C. Diffusion impairment
D. R-L shunt
(Problem Solving) - Respiratory
C - diffusion impairment
In pulmonary edema there is fluid that acts as a barrier between the alveolo-capillary
membrane which increases the travel time of the O2 from the alveoli to the capillary thus hindering the
diffusion of the gas through the membrane and subsequently lowers theO2 levels in the circulation.
V/Q mismatch as a cause of hypoxemia occurs in two stages: V ventilation when there
will be less O2 delivered to the alveoli due to airflow obstruction as it happens in pneumonia Q (perfusion)
hypoxemia occurs despite adequate oxygenation if the circulation is blocked as in pulmonary embolism
Hypoventilation – could cause low O level due to decreased amount of O2 delivered in to
the alveoli due to central causes – (CNS depression or infection) or due to low levels of O2 in the
atmosphere due to high altitude
R-L shunt this happens particularly in patient with cardiac shunts where a high
percentage of the cardiac output returns to the general circulation without passing through the lungs. Or
this can occur in cases of intrapulmonary shunts as well
(Nelson’s Textbook of Pediatrics, 17th ed, 2004)

10
48. The common infectious cause/s of bronchiectasis is/are:
A. Pertussis
B. Klebsiella pneumoniae
C. Streptotoccus pneumoniae
D. H. influenzae
(Recall) - Respiratory
A - Pertussis
Infections due to Bordatella pertussis, measles, rubella, togavirus, respiratory syncytial virus and
Mycobacterium tuberculosis induce chronic inflammation, progressive bronchial wall damage and
dilatation of the bronchial tree. The common thread in the pathogenesis of bronchiectasis is difficulty
clearing secretions and recurrent infections.
(Nelson’s Textbook of Pediatrics, 17th ed, 2004)

49. The pathologic findings of bronchopulmonary dysplasia (BPD) consist of the following:
A. Decreased alveolarization
B. Decreased alveolar septation
C. Minimal airway disease
D. All of the above
(Recall ) - Respiratory
D - all of the above
BPD is a result of lung injury in infants requiring mechanical ventilation and
supplemental oxygenation. It is apparent that patients with BPD have decreased alveolarization, alveolar
septation and minimal airway disease all of which suggest arrest in lung development. The lung injury
occurring in children is due to an interaction of multiple factors. Since RDS is a disease of progressive
alveolar collapse, Atelectasis which is affected by insufficient PEEP together with ventilator-induced
increased lung volume and regional overdistention promotes injury. Oxygen promotes injury by producing
free radical that cannot be metabolized by immature antioxidant systems. Therefore, mechanical
ventilation and /or oxygen injure the preterm lung by affecting alveolar and vascular development.
Moreover, inflammation as measured by circulating neutrophils and macrophage in the alveolar fluid and
pro-inflammatory cytokines contribute to the progression of the lung injury.
(Nelson’s Textbook of Pediatrics, 17th ed, 2004)

50. A patient is considered to have intermittent asthma when the following is/are present:
A. PEFR variability = <20%
B. PEFR 60-79% of predicted
C. FEVI <60%
D. PEFR variability = 20-30%
(Problem Solving) - Respiratory
A - PEFR variability =<20% Peak expiratory flow rate variability is a measure of the stability of the
airways and it is considered as a diagnostic tool to predict the success of one’s treatment PEFR
variability =<20% is still within normal limits. Intermittent asthma has normal PEFR and PEFR
variability values. Symptoms of these patients are very infrequent
(B) Once PEFR is 60-79% of the predicted, it only means that airway obstruction is present [(N)
PEFR = ≥ 80% of the predicted] and signals that the patient belongs to the moderate persistent
category
(C ) FEV1 (forced expiratory volume in one second) is a measure of airflow obstruction. And the
value of <60% indicates that the patient belongs to the category of severe persistent asthma.
(D) PEFR variability =20-30% means that airways are still unstable and asthma is not well
controlled and the patient belongs to the mild persistent asthma category
(Nelson’s Textbook of Pediatrics, 17th ed, 2004)

51. An 8-year old presents with sneezing, clear rhinorrhea, and nasal itching. P.E. findings show boggy,
pale nasal edema with a clear discharge. The most likely diagnosis is:
A. Foreign body
B. Vasomotor rhinitis
C. Neutrophilic rhinitis
D. Allergic rhinitis
(Problem Solving) – Immunology/Allergology
D - Allergic rhinitis is often seasonal and associated with allergic conjunctivitis. Eosinophils
predominate in the nasal secretions.
(Nelson’s Textbook of Pediatrics, 17thed, 2004, pp. 759-760)

52. The mother of 7-year old girl with acute strep throat calls to report that within 15 minutes after the
first dose of oral penicillin you prescribed, she is complaining of itching and has developed hives. Which
of the following should you recommend?
A. Give her oral antihistamines and call again if not improved within 30 minutes
B. Bring her to your office or the nearest emergency room

11
C. Substitute erythromycin for penicillin
D. Bring her to the nearest emergency room once difficulty of breathing is experienced
(Problem Solving) – Immunology/Allergology
B - The urticarial reaction described in the question may develop into anaphylaxis which requires
emergency treatment. Aside from this, penicillin should be stopped and a substitute non-penicillin
appropriate antibiotic chosen.
(Nelson’s Textbook of Pediatrics, 17th ed, 2004,Chap 137, pp. 781-782)

53. A child has abdominal pain, arthritis, microscopic hematuria, and a purpuric rash only on the lower
extremities. Which of the following is the most likely diagnosis?
A. Meningococcemia
B. Varicella
C. Henoch-Schonlein vasculitis
D. Post streptococcal glomerulonephritis
(Problem Solving) – Immunology/Allergology
C -The purpura on the lower extremities suggests Henoch-Schonlein Vasculitis.
Meningococcemia is generalized. Varicella gives papulo-vesicular lesions which are likewise
generalized. Erythema nodosum is the cutaneous lesion usually found on the lower extremities in post-
strep infection.
(Nelson’s Textbook of Pediatrics, 17th ed, 2004,pp. 794, 826-828)

54. A 2-day old neonate with vomiting of bilous material since birth was brought to your hospital. X-rays
taken showed “double-bubble”.
A. Metabolic problems must be addressed initially
B. Schedule for emergency laparotomy
C. Do upper GI series
D. Intubate once seen
(Problem Solving) - Neonatology
A - (Nelson’s Textbook of Pediatrics,, 17th ed, 2004, pp. 1233-1234)

55. A 24-hour old neonate is brought to the ER because of inability to pass meconium in 24 hours. He
does not have vomiting or distention. Your recommendation would be to:
A. Do suction rectal biopsy
B. Request for barium enema
C. Request for abdominal x-ray
D. Observe the patient
(Problem Solving) - Neonatology
D - (Nelson’s Textbook of Pediatrics, 17th ed, 2004, pp. 1232-1241)

56. A 7-day old 900 gram pre-term has been noted to have abdominal distention with gastric retention.
OGT drainage is 10 cc in 24 hours and stool occult blood is positive. Abdominal x-ray showed
pneumatosis intestinalis. This patient should:
A. Undergo immediate surgery
B. Have intensive medical therapy
C. Peritoneal drainage
D. Have a blood culture.
(Problem Solving) - Neonatology
B - (Nelson’s Textbook of Pediatrics, 17th ed, 2004, pp. 590-591)

57. A live 30 weeks of gestation baby boy was born via cesarean section to a diabetic mother. Grunting
and tachypnea was noted on the 6th hour of life. After receiving therapeutic measures, the patient
improved. However on the 4th day of life, lethargy, apnea and poor muscle tone was noted.
Transfontanel cranial ultrasonography was done and showed increased echogenecity at the
thalamocapsular region with ventricular dilatation. What is the grade of this patient’s germinal matrix
hemorrhage?
A. Grade 1
B. Grade 2
C. Grade 3
D. Grade 4
(Problem Solving) - Neonatology
D - (Nelson’s Textbook of Pediatrics, 17th ed, 2004, pp. 563)

58. Which of the following chest radiographic findings can be found in mild ventricular septal defect?
A. Small heart
B. Increased pulmonary vascularity
C. Increased size of the aorta
D. Left atrial enlargement

12
(Problem Solving) - Neonatology
B - (Nelson’s Textbook of Pediatrics, 17th ed, 2004, pp. 1509)

59. Meconium aspiration in utero is explained by:


A. Obstruction of fetal airways
B. Chemical property of meconium
C. Intrauterine infection
D. Chronic fetal asphyxia
(Problem Solving) - Neonatology
D - (Nelson’s Textbook of Pediatrics, 17th ed, 2004, pp. 547, 583-584)

60. A key predictor for death or brain damage after an asphyxial episode:
A. HIE Sarnat Stage 2
B. Failure to establish spontaneous respiration by 15 minutes
C. Onset of seizure within the first 24 hours of life
D. Establishment of adequate oral feedings by 48 days of life
(Recall) - Neonatology
B - (Nelson’s Textbook of Pediatrics, 17th ed, 2004, pp. 567)

61. In a newborn suspected of having choanal atresia, respiratory distress may be relieved by:
A. Opening the mouth
B. Intubation
C. Bag and mask ventilation
D. Administering O2
(Problem Solving) - Neonatology
A - (Nelson’s Textbook of Pediatrics, 17th ed, 2004, pp. 1387)

62. The most common clinical manifestation of Persistent Pulmonary Hypertension is:
A. Respiratory distress
B. Pallor
C. Cyanosis
D. Apnea
(Recall) - Neonatology
C - (Nelson’s Textbook of Pediatrics, 17th ed, 2004, pp. 585)

63. Which of the following statements regarding the diagnosis of intrauterine infection is/are accurate?
A. IgM in neonatal serum may be used as a screening tool
B. Total IgM has a low rate of both false positive and false negative results
C. IgM titers may have low specificity and low sensitivity
D. IgG rising titers in infancy are not helpful
(Problem Solving) - Neonatology
A- (Nelson Textbook of Pediatrics, 17th ed, 2004)
64. Thyrotoxicosis in the first day of life is most likely to occur in an infant born to a mother
A. With untreated hypothyroidism
B. With untreated Grave’s disease
C. With Grave’s disease being treated with antithyroid medications
D. Receiving iodides as therapy for chronic Thyrotoxicosis
(Problem Solving) - Neonatology
B - (Nelson’s Textbook of Pediatrics, 17th ed, 2004, pp. 1886)

65. A 6-year old male was brought to the OPD because of jaundice of 5 days associated with anorexia of
one week. A hepatitis profile done on him revealed the following:
Anti-HAV (IgM) - Reactive
HBsAg - Non-reactive
Anti-HBc - Non-reactive
Anti-HBe - Non-reactive
Anti-HBs - Non-reactive
The patient Had a recent:
A. HAV and HBV infection
B. HAV infection with a post infection of HBV
C. HAV infection with HBV immunity
D. HAV infection and post infection of HAV
(Problem Solving) – Infectious Diseases
C – (Nelson’s Textbook of Pediatrics, 17th ed., 2004, pp. 1324-1329)

66. A 9-month old female was brought to the OPD because of watery diarrhea, yellowish, non-bloody,
non-mucoid stools, with no pus nor RBC’s. Her anterior fontanel is slightly sunken. The most likely
organism to cause this type of diarrhea is:

13
A. ETEC
B. Vibrio cholera
C. Rotavirus
D. Norwalk virus
(Problem Solving) – Infectious Diseases
C – (Nelson’s Textbook of Pediatrics, 17th ed., 2004, pp. 1324-1329)

67. Which of the following is TRUE of candidal infection?


A. Diaper dermatitis is the most common infection caused by candida
B. With improved survival of very LBW infants, candidemia has become less frequent n
NICU’s
C. Fluconazole is the drug of choice for the treatment of systemic candiciasis
D. Most cases of candidemia is Immunocompromised patients are due to non-candida
albicans spp.
(Problem Solving) – Infectious Diseases
A - (Nelson’s Textbook of Pediatrics, 17th ed., 2004, pp. 1012-1013)

68. Which of the following statements is NOT TRUE of Amebiasis?


A. The infective stage is the Entamoeba histolytica cyst
B. The pathogenic stage is the Entamoeba histolytica trophozoite form
C. A carrier of E. histolytica cysts should be treated
D. It is the most common cause of bloody stools
(Recall) – Infectious Diseases
D - (Ref. Nelson’s Textbook of Pediatrics, 17th ed., 2004, pp. 1123-1125)

69. Which of the following diseases has the greatest capacity to be a “pandemic?”
A. HIV
B. Influenza
C. Measles
D. Hepatitis B

(Recall) – Infectious Diseases


B - (Nelson’s Textbook of Pediatrics, 17th ed., 2004, pp. 1072-1074)

70. A newborn was found to have the following anomalies: hydrocephalus, cicatricial scarring over the
6th-7th left intercostals, malformed feet (fusion and maldevelopment of both feet). Which congenital
infection shows these findings at birth?
A. HIV
B. CMV
C. Parovovirus B19
D. Varicella-Zoster virus
(Recall) – Infectious Diseases
D - (Nelson’s Textbook of Pediatrics, 17th ed., 2004, p. 1059)

71. Which of the following features is TRUE of all infants born to HIV-infected women?
A. They will have low CD4 cell counts
B. They will eventually develop AIDS
C. They will have antibodies to HIV
D. They will be infected with HIV
(Problem Solving) – Infectious Diseases
C - (Nelson’s Textbook of Pediatrics, 17th ed., 2004, p. 1161)

72. One of the following is LEAST associated with ascaris infection in humans:
A. Intestinal obstruction
B. Loeffler’s syndrome
C. Iron deficiency anemia
D. Pancreatitis
(Recall) – Infectious Diseases
C - (Nelson’s Textbook of Pediatrics, 17th ed., 2004, pp. 1156)

73. Which of the following does NOT RESULT from a vesicoureteral reflux?
A. Hypertension
B. Chronic failure
C. Proteinuria
D. Dilatation of the ureters
(Problem Solving) – Nephro/GU
C - Vesicoureteral reflux results from valvular incompetence at the uretero vesicular junction as a
result of a shortened segment of ureter within the bladder wall. It is often associated with other

14
genitourinary anomalies. Vesicoureteral reflux can result in chronic renal failure, dilatation of the ureters,
hypertension and urinary tract infections. Proteinuria results from glomerular injury.
(Nelson’s Textbook of Pediatrics, 17th ed, 2004,pp. 1790-1794)

74. The triad of microangiopathic hemolytic anemia, renal failure, and thrombocytopenia is characteristic
of which one of the following?
A. Membranous lupus nephritis
B. Focal glomerulonephritis secondary to sepsis
C. Acute post-streptococcal glomerulonephritis
D. Hemolytic-uremic syndrome
(Recall) – Nephro/GU
D -The name gives the answer away.
(Nelson’s Textbook of Pediatrics, 17th ed, 2004,pp. 1746-1747)

75. A 2-year old male developed an upper respiratory tract infection that was followed in 2 weeks by
general edema. His blood pressure is normal. Urinalysis reveals 2-5/hpf and +4 protein. His BUN is 19
mg/dl, creatinine 0.6 mg/dl, cholesterol 402 mg/dl, serum albumin 0.9 g/dl, ASO=200, and C3=92 mg/dl.
The most likely diagnosis would be:
A. Poststreptococcal glomerulonephritis
B. Membranous glomerulonephritis
C. Minimal lesion nephrotic syndrome
D. Focal sclerosis
(Problem Solving) – Nephro/GU
C - Hypoalbuminemia, proteinuria, edema and hyperlipidemia constitute the nephrotic syndrome.
Hypertension, azotemia, edema or hematuria would suggest nephritis but may also be encountered in
minimal lesion nephrotic syndrome. This patient has nephrotic syndrome, not nephritis.
(Nelson’s Textbook of Pediatrics, 17thed, 2004,pp. 1740-1746 ; 1753-1757)

76. The recommended age to perform corrective surgery in a child with a unilateral undescended testes
is:
A. The first 6 months of life
B. Between 21 and 18 months of age
C. Between 5 and 7 years of age
D. Before puberty
(Recall) – Nephro/GU
B - Corrective surgery performed between 12 and 18 months of age represents a safe balance
between anesthetic risk, allowance of time for the testes to descend, and the risks of leaving a testis in
the abdomen.
(Nelson’s Textbook of Pediatrics, 17thed, 2004,pp. 1817-1820)

77. Maternal varicella results in severe neonatal varicella when maternal infection takes place during:
A. The 1st trimester
B. The 2nd trimester
C. The 3rd trimester
D. The week before and after delivery
(Problem Solving) – Infectious Diseases
D - Birth within 1 week before or after the onset of maternal varicella frequently results in the
newborn developing varicella, which may be severe. The risk to the newborn is dependent on the
amount of maternal anti-VZV antibody that the fetus acquired transplacentally before birth. If the internal
between maternal chickenpox and parturition is less than 1 week, the newborn will be unlikely to have
protective VZV antibody and neonatal chickenpox may be exceptionally severe.
(Nelson’s Textbook of Pediatrics, 17th ed., 2004, p. 1058)

78. A 2-year old has a positive tuberculin test (15 mm induration). Which of the following would be
suggestive of “military tuberculosis?”
A. Fever
B. Hepatosplenomegaly
C. Hilar adenopathy on chest x-ray
D. Cough
(Problem Solving) – Infectious Diseases
B - “Miliary tuberculosis” suggests lymphohematogenous spread or disseminated form of TB,
occurring is distant sites, including liver, spleen, skin and other organs aside from the lungs. Fever
and cough are non-specific manifestations of TB which may be found in other diseases, while hilar
adenopathy on chest x-ray, may be found in primary pulmonary TB. (Nelson’s Textbook of Pediatrics, pp.
962-964)

79. A 3-year old nonimmunized child is seen at the OPD and diagnosed as having measles. There is an
8-month old nonimmunized sibling at home. Appropriate management of this sibling would include:

15
A. A modifying dose of gammaglobulin
B. A preventive dose of gammaglobulin
C. Immediate immunization with live attenuated measles vaccine
D. Immediate immunization with killed measles vaccine
(Problem Solving) – Infectious Diseases
A - Passive immunization with immune globulin is effective for prevention and attenuation of
measles within 6 days of exposure. Susceptible household and hospital contacts who are <12 months of
age or who are pregnant should receive immune globulin (modifying dose, 0.25 ml/kg; maximum 15 ml)
IM as soon as possible after exposure, but within 5 days. Immunocompromised persons should receive
immune globulin (preventive dose, 0.5 ml/kg ; maximum 15 ml) IM regardless of immunization status.

80. An 8-year old male consulted the OPD because of high grade fever and sore throat. The pertinent
P.E. findings were: hyperemic oropharynx, enlarged tonsils with exudates, petecchiae on the soft palate
and painful, enlarged cervical lymphadenopathies. This patient is most likely suffering from:
A. Streptococcus pyogenes
B. Epstein Barr Virus
C. Adenovirus
D. Corynebacterium diphtheria
(Recall) – Infectious Diseases
C - (Nelson’s Textbook of Pediatrics, 17th ed., 2004, pp. 870-879)

81. An 8-month old boy presents with failure to thrive, thrush, lymphadenopathy, and pneumocystis carinii
pneumonia. He most likely has:
A. Severe malnutrition
B. Acute leukemia
C. HIV infection
D. X-linked hypogammaglobulinemia
(Recall) - Infectious Diseases
C - These constellation of manifestation are associated with HIV infection. The clinical
manifestations of HIV infection vary widely among infants, children, and adolescents. In most infants,
PE findings at birth are normal. Initial symptoms are subtle, such as lymphadenopathy and
hepatosplenomegaly, or non-specific such as failure to thrive, chronic or recurrent diarrhea, interstitial
pneumonia, or oral thrush, and may be distinguishable only by their persistence. Whereas systemic and
pulmonary findings are common in the United States and Europe, chronic diarrhea,wasting, and severe
malnutrition predominate in Africa. Symptoms found more commonly in children than adults with HIV
infection include recurrent bacterial infections, chronic parotid swelling, lymphocytic interstitial
pneumonitis (LIP), and early onset of progressive neurologic deterioration. The HIV classification system
is used to categorize the stage of pediatric disease by using 2 parameters: clinical status, and degree of
immunologic impairment (absolute CD4 lymphocyte count or the percentage of CD4 cells).
Category A (Mild Symptoms):
Children with at least 2 mild symptoms such as:
- lymphadenopathy
- parotitis
- hepatomegaly
- splenomegaly
- dermatitis
- recurrent or persistent sinusitis or otitis media
Category B (Moderate Symptoms):
- lymphocytic interstitial pneumonitis (LIP)
- oropharyngeal thrush persisting for <2 months
- recurrent or chronic diarrhea
- persistent fever >1 month
- hepatitis
- recurrent herpes simplex stomatitis or HSV esophagitis or pneumonitis
- disseminated varicella (i.e., with visceral involvement)
- cardiomegaly
- nephropathy
Category C (Severe Symptoms):
Children with 2 serious infections (i.e., sepsis, meningitis, pneumonia) in a 2 year period
- esophageal or lower respiratory tract candidiasis
- cryptococcosis
- cryptosporidiosis (>1 mo)
- encephalopathy
- malignancies
- disseminated myocobacterial infection
- pneumocystis carinii pneumonia (PCP)
- cerebral toxoplasmosis (onset after 1 month of age)
- severe weight loss

16
The Pneumocystis carinii pneumonia (PCP) is the most common opportunistic infection
in the pediatric population. The peak incidence of PCP occurs at age 3-6 months with the highest
mortality rate in children <1 year of age.
(Nelson’s Textbook of Pediatrics, 17th ed., 2004, pp. 1112-1113)

82. In our country, the first dose of live attenuated measles vaccine should be administered:
A. at 4 months of age
B. at 6-9 months of age
C. at 12-15 months of age
D at 18-24 months of age
(Recall) - Infectious Diseases
B - The attack rate of measles among the susceptibles in <1 year is 80% (DOH Philippines
2000), such that even if vaccine efficacy at 6 months is just 50% and at 9 months it is
85%, we give it as early as 6 months, during which time transplacentally acquired
maternal antibodies for measles would already be at its nadir.
(DOH Philippines 2000)

83. Neonatal bacterial sepsis is most commonly caused by which one of the following organisms (in the
Philippines):
A. Group B streptococci
B. Pseudomonas aerugenosa
C. Streptococcus pneumoniae
D. E. coli
(Recall) - Infectious Diseases
D - In our country, gram (-) organisms are the most common causes of neonatal bacterial sepsis
(2:1) compared with the gram positive organisms. The gram negative enteric bacilli
like E. coli are the more common causative agents. Pseudomonas
aerugenosa, a gram negative bacilli, is a common nosocomial pathogen.
(Textbook of Pediatrics and Child Health, Del Mundo, Fe et. al (eds.) 4 th ed, 2000, p. 265)

84. The first clinical manifestation of tetanus neonatorum usually is:


A. Fever
B. Vomiting
C. Spasms
D. Difficulty sucking and swallowing
(Recall) - Infectious Diseases
D - Neonatal tetanus is generalized in type and starts as progressive difficulty in sucking and
irritability.
(Textbook of Pediatrics and Child Health, Del Mundo, Fe, et al (eds.), 4 th ed, 2000, p. 469)

85. The usual course of pertussis in an infant is characterized by:


A. 4 – 5 days of high grade fever followed by cough and whooping
B. sudden onset of fever, cough, and whooping
C. rhinitis and possibly low grade fever, followed by gradual worsening of cough and
finally whooping
D. gradual onset of cough, followed by abrupt onset of fever and whooping
(Recall) - Infectious Diseases
C - Pertussis is a 6-week disease, divided into catarrhal (congestion, rhinorrhea, then low grade
fever, lacrimation, conjunctival suffusion), paroxysmal, and convalescent stages.
(Nelson’s Textbook of Pediatrics, 17th ed., 2004, p. 909)

86. The findings of sudden onset of fever, petecchial rash, and BP = 70/50 in a 5 year old child is most
suggestive of infection with:
A. Neisseria meningitidis
B. Hemophilus influenzae
C. Staphylococcus aureus
D. Streptococcus pneumoniae
(Problem Solving) - Infectious Diseasess
A - These findings are compatible with meningococcemia caused by Neisseria meningitides
which can progress rapidly over hours to septic shock. The other etiologic agents,
while they may cause a similar picture these are usually in relation to DIC which would
manifest over a longer period of time, not in a matter of hours.
(Nelson’s Textbook of Pediatrics, 17th ed, 2004, p. 897)

87. Of the following parasitic infections, which is most likely to present with intestinal obstruction?
A. Trichuris trichura
B. Necator americanus
C. Ascaris lumbricoides

17
D. Enterobius vermicularis
(Recall) - Infectious Diseases
C - A large mass of Ascaris lumbricoides leads to intestinal obstruction. Rectal prolapse is
associated with Trichuris trichura. Heavily infected children with Necator
americanus suffer from intestinal blood loss resulting in iron deficiency, which can
lead to anemia as well as protein malnutrition. Pruritus ani is associated with
enterobiasis.
(Nelson’s Textbook of Pediatrics, 17th ed., 2004, pp. 1156-1159)

88. A 10-month old child has a temperature of 39-40’C for 4 days without other signs. On the 4 th day, a
maculopapular rash appears, and the temperature returns to normal. The most likely diagnosis is:
A. scrub typhus
B. roseola
C. rubeola
D. echoviral infection
(Recall ) - Infectious Diseases
B - (Nelson’s Textbook of Pediatrics, 17th ed., 2004, pp. 1069-1072)

89. Which of the following is the chemoprophylactic antimicrobial given to intimate contacts of a 7-year
old with meningococcal meningitis?
A. Rifampin
B. aqueous Pen G
C. Isoniazid
D. Erythromycin
(Problem Solving) - Infectious Diseases
A - (Nelson’s Textbook of Pediatrics, 17th ed., 2004, pp. 898-899)

90. The clinical manifestations in DHF are secondary to the pathogenetic mechanism of:
A. direct invasion of the virus to the different organ system
B. hypersensitivity reaction
C. immune enhancement
D. adherence of the viruses to the endothelial cells
(Recall) - Infectious Diseases
C - (Textbook of Pediatrics & Child Health, Del Mundo, et al (eds.), 4 th ed, 2000, p. 561)

91. In a case of DHF (Grade 3) who is bleeding profusely, which of the following fluids (all available)
would you give?
A. properly typed & X-matched fresh whole blood
B. properly typed & X-matched fresh plasma
C. D5LRS
D. D5NSS
(Problem Solving) - Infectious Diseases
A - (Textbook of Pediatrics & Child Health, Del Mundo, et al (eds.), 4 th ed, 2000, p. 569 - 571)

92. The following CSF analysis results are compatible with which of the following clinical entities?
CSF Results: Opening pressure = 300 mm H2O
WBC = 296 (segs: 10% ; lymphos : 90%)
Protein = 2 g/L
CSF sugar / RBS = 20%
A. Acute meningococcal meningitis
B. TB meningitis
C. Japanese B encephalitis
D. Febrile seizures
(Problem Solving) – Infectious Diseases
B - (Nelson’s Textbook of Pediatrics, 17th ed., 2004, pp. 965, 2040-2044)

93. On the 3rd day of treatment for Hemophilus influenzae meningitis, an eight month old child who had
been alert is noted to be lethargic. Serum electrolytes reveal the following:
Na = 120 mEq/L
Cl = 83 mEq/L
K = 3.1 mEq/L
BUN = 2 mg/dl
The most likely cause of the lethargy and hyponatremia in this patient:
A. Acute renal failure
B. Congestive heart failure
C. Syndrome of inappropriate ADH secretion
D. Subdural effusions

18
(Problem Solving) – Infectious Diseases
C - (Nelson’s Textbook of Pediatrics, 17th Ed, 2004, p. 200)

94. A 28 year old primigravida mother, who had a past history of adequately treated PTB, gave birth to a
healthy 3 kg baby. Which of the following preventive measures against TB would you take?
A. Separate the baby from the mother for another month
B. Separate the baby from the mother and give INH for 3 months
C. Give BCG only
D. Give BCG now but separate the baby from the mother for a month
(Problem Solving) – Infectious Diseases
C - (Textbook of Pediatrics & Child Health, Del Mundo, et al (eds.), 2000 pp. 516-525)

95. A 2-year old boy was admitted because of low to moderate grade fever of 3 weeks, on and off frontal
headache of 1 week, squinting of 1 day, one episode of generalized seizure of 2 minute duration 6 hours
prior to admission. No medical consult done. No medications given except paracetamol. Which of the
following clinical consideration is NOT COMPATIBLE with this history?
A. Acute meningococcal meningitis
B. TB meningitis
C. Cryptococcal meningitis
D. Brain abscess of otogenic origin
(Problem Solving) – Infectious Diseases
A - (Nelson’s Textbook of Pediatrics, 17th ed., 2004, pp. 965, 2040-2044)

For Nos. 96 to 100:


Mario is 2 ½ years old, brought to the Well Child Clinic for his regular visit. On physical exam, he
had the following anthropometric measurements: Weight - 15 kgs, Height – 90 cms, Head
Circumference – 48 cms. On developmental screening, he was able to do the following: could
stand momentarily on one foot, draw a circle and imitate a cross, talk in simple sentences
and tell a story. Mother claimed that he is dry by night.

96. Describe the nutritional status of Mario:


A. not stunted, not wasted
B. mildly stunted, not wasted
C. not stunted, mildly wasted
D. Mildly stunted, mildly wasted
(Problem Solving) - Neurodev
A - (Textbook of Pediatrics & Child Health, Del Mundo, et al (eds.), 4 th ed, 2000, pp. 78-80)

97. The head circumference of Mario is:


A. normal
B. below normal
C. above normal
D. variation of normal
(Problem Solving) – Neurodev
A - (Textbook of Pediatrics & Child Health, Del Mundo, et al (eds.), 4 th ed, 2000, p. 80)

98. Based on developmental screening, his development is:


A. normal
B. delayed
C. advanced
D. deviant
(Problem Solving) - Neurodev
C - (Textbook of Pediatrics & Child Health, Del Mundo, et al (eds.), 4 th ed, 2000, pp. 83-112)

99. The approximate developmental age of Mario is:


A. 2 years old
B. 2 ½ years old
C. 3 years old
D. 3 ½ years old
(Problem Solving) – Neurodev
C - (Textbook of Pediatrics & Child Health, Del Mundo, et al (eds.), 4 th ed, 2000, pp. 83-112)

100. At 2 years of age, Mario had a height of 85 cms. How tall would he be when he becomes an adult?
A. 4 feet 6 inches
B. 5 feet
C. 5 feet 6 inches
D. 6 feet
(Recall) - Neurodev

19
C - (Textbook of Pediatrics & Child Health, Del Mundo, et al (eds.), 4 th ed, 2000, pp 78-80)

Subject: Pediatrics
Instructions: Choose the BEST answer

1. The parameters used to estimate the gestational age based on physical maturity are description of
the following EXCEPT:
A. lanugo
B. plantar surface
C. genitalia
D. pupillary dilatation

Ans: D

2. An infant weighing 1400 gm is born at 32 weeks gestation in a delivery room that has an ambient
temperature of 24 degrees centigrade. Within a few minutes of birth, this infant is likely to exhibit all
the following EXCEPT:
A. Pallor
B. Shivering
C. a fall in body temperature
D. metabolic acidosis

Ans: B

3. The immediate postnatal changes in a term newborn includes the following EXCEPT
A. decrease in pulmonary vascular resistance
B. decrease in right to left shunting via ductus arteriosus
C. increase in venous return to the left atrium
D. increase right to left shunting via foramen ovale

Ans: D

4. An infant has the following findings at 5 minutes of life; pulse 130 per minute, cyanotic hands and
feet, good muscle tone, and a strong cry. This infant’s Apgar score is
A. 7
B. 8
C. 9
D. 10

Ans: C
5. A newborn infant was noted to have the following physical features at birth: Weight 3.0 kg, absent
lanugo, white parchment-like desquamating skin and long nails. The infant’s gestational age is most
likely :
A. >42 wks
B. 37 – 39 wks
C. <37 wks
D. 40-42 wks

Ans: A

6. Which of the following patterns noted on continuous monitoring of fetal heart rate is most indicative of
fetal distress?
A. Baseline variability with periodic acceleration
B. Increasing baseline variability
C. Early deceleration pattern
D. Late deceleration without baseline variability

Ans: D

7. Maternal condition that would post for high risk pregnancy


A. Pregnancy interval of 2-3 years
B. Maternal age 20-25 year old
C. Primigravid status
D. Inadequate pre-natal care

Ans: D

20
8. Anticipated neonatal morbidities associated with maternal risk factors include
A. Neonatal macrosomia with Maternal Diabetes
B. Neonatal euthyroid state with Maternal Graves disease
C. Neonatal thrombocytosis with Maternal thrombocytopenia
D. Neonatal polycythemia with Maternal placenta previa

Ans: A

9. A mother delivers a neonate with meconium staining and low Apgar scores of 3 at 1 and 5 mins. Of
life. The goals of resuscitation are the following EXCEPT
A. Minimize heat loss
B. Establish normal respiration and lung expansion
C. Support cardiac output
D. Anticipate neonatal demise

Ans: D

10. A newborn was noted to have recurrent episode of aspiration with excessive salivation, most likely the
mother had
A. Polyhydramnios
B. Oligohydramnios
C. Normohydramnios
D. Anhydramnios

Ans: A

11. Which of the following blood factors are decreased in the newborn?
A. VII, IX, and X
B. II, V, VII, and IX
C. V, VII, IX, and X
D. VII, VII, IX, and XI

Ans: B

12. Which situation is Jaundice most likely physiologic in a term infant?


A. Jaundice at 12 hours of age
B. Serum bilirubin increasing by 5 mg/dl/24 hours or less in the first 2 to 4 days
C. Direct (conjugated) serum bilirubin greater than 1 mg/dl
D. Jaundice at 12 days of age
Ans: B

13. The most appropriate treatment for hyperbilirubinemia (11.2 mg/dl) in a 3-week-old breast-fed infant
with normal growth and development?
A. Phototherapy
B. Exchange transfusion
C. Phenobarbital
D. None of the above

Ans: D

14. A primiparous woman whose blood type is O-positive gives birth at term to an infant who has an A-
positive blood and a hematocrit of 55%. A serum bilirubin level obtained at 36 hours of age is 12
mg/100 ml. Which of the following laboratory findings would be most characteristic of this infant’s
disease?
A. An elevated reticulocyte count
B. A weakly positive direct Coombs test
C. Nucleated red blood cells in the blood smear
D. Hematocrit less than 55%
Ans: D

15. A full term infant was born by normal spontaneous delivery. Nursery stay was unremarkable and
baby was breastfed. On the 2nd week of life, baby was noted to be jaundiced. This baby is most likely
having:
A. Physiologic jaundice
B. Pathologic jaundice
C. Breast-milk jaundice
D. Kernicterus

21
Ans: C

16. Typical abdominal x-ray in necrotizing enterocolitis:


A. “double-bubble” sign
B. “string” sign
C. Pneumatosis intestinalis
D. “apple peel” sign

Ans: C

17. A fecaloid vomitus indicates


A. Obstruction to the stomach
B. Obstruction in the duodenum proximal to the ampulla of Vater
C. Narrowing or closure of the intestinal lumen distal to the ampulla of Vater
D. Obstruction low in the intestinal tract

Ans: D

18. TRUE about necrotizing enterocolitis:


A. It is primarily a disease of infants 6-12 months old.
B. Breast milk feedings are not protective.
C. Bloody stools are seen in most patients.
D. It usually presents with abdominal distention with gastric retention.

Ans: D

19. A two-week-old premature infant is found to have several milliliters of formula still present is the
stomach two hours after being fed. Also noted are gastric distention and the passage of blood-
streaked stools. Which historical factor would best support a tentative diagnosis of necrotizing
enterocolitis?
A. Passage of a thick tenacious meconium plug at 24 hours of age
B. Severe hyaline membrane disease with anoxic episodes in the first week of life
C. A maternal history of severe ulcerative colitis
D. A history of milk-protein allergy in family members

Ans: B

20. A woman was noted to have a large volume of amniotic fluid at the time of her delivery of her child. At
6 hours of age, her baby begins regurgitating small amounts of mucus and bile-stained fluid. P.E. is
normal. Abdominal x-ray obtained showed a “double-bubble” sign. The most likely diagnosis of this
infant’s disorder:
A. Esophageal atresia
B. Pyloric stenosis
C. Midgut volvulus
D. Duodenal atresia

Ans: D

21. A 1 month old infant forces at near object attains its visual activity of 20/20 at age
A. 2
B. 3
C. 4
D. 5

Ans: C

22. A 6 month old child who weighed 3.0 kg at birth would have a weight of
A. 5 kg
B. 7 kg
C. 9 kg
D. 11 kg

Formula used:

Age in months + 9
------------------------
2

22
6+9 15
-------- = ---------- = 7.5 kg
2 2

Ans: B

23. At age 15 months, which motor activity can be performed normally?


A. Walks upstairs with alternating steps
B. Climbs the stairs one step at a time
C. Walks alone by herself
D. Runs and seldom falls

Ans: C

24. At 12 months, a child can already


A. Say bye-bye
B. Say his full name
C. Comprehend words as “hello”
D. Utter 2 words like the “Dada” and “Mama”
Ans: D

25. The first visible sign of puberty in boys.


A. growth of pubic hair
B. enlargement of the penis
C. testicular enlargement
D. axillary perspiration

Ans: C

26. The sexual maturity of a girl whose breast buds appeared when she was 10 years old and had
menarche at 16 years old.
A. normal
B. precocious
C. delayed
D. undetermined
Ans: C
If a girl has no breast buds by the age of 13 years, or if more than 5 years separate the onset of
pubertal change from menarche, her puberty is delayed.

27. The most common stimulus in the gut provoking abdominal pain:
A. edema
B. ischemia
C. tension or stretching
D. accumulation of tissue metabolites

Ans: C

28. A 6 month-old male infant previously well, suddenly developed paroxysmal colicky abdominal pain at
frequent intervals accompanied by straining efforts is most likely having this condition.
A. Acute Appendicitis
B. Abdominal Epilepsy
C. Urinary Tract Infection
D. Intussusception

Ans: D
Paroxysmal colicky abdominal pain at frequent intervals in a previously well infant is characteristic
of Intussusception.

29. The most likely diagnosis of a 9 year old male with severe epigastric pain and muscle rigidity on the
epigastrium, unrelieved by ordinary antispasmodic.
A. Urolithiasis
B. Abdominal Epilepsy
C. Acute Pancreatitis
D. Acute Hepatitis
Ans: C

23
The pain in Urolithiasis may also be severe but is usually located at the lumbar and lower back
areas. The pain in abdominal epilepsy is usually in the periumbilical area. Abdominal pain in
Acute Hepatitis is usually not severe. Acute Pancreatitis is characterized by severe epigastric
pain and muscle rigidity.

30. The recommended daily allowance of Protein during the 1st 6 months of life is about:
A. 1.0 g/kg/24 hr
B. 2.0 g/kg/24 hr
C. 3.0 g/kg/24 hr
D. 4.0 g/kg/24 hr
Ans: B
The current RDA for protein during the first 6 months of life in a healthy term infant is 2.0-2.2
g/kg/24 hour.

31. Breastmilk from mothers whose diet is sufficient and balanced supply all the necessary nutrients
needed by the infant. One of the following is found to be insufficient at birth:
A. Vitamin A
B. Vitamin D
C. Fluoride
D. Iron

Ans: C
Amount of iron and Vitamin D are sufficient during the 1 st 4-6 months of life. Supplement is
needed beyond this period. If the water supply is not adequately fluoridated (< 0.3ppm), the
breastfed infant should receive at least 10 ug of fluoride daily for the 1 st 6 months of life.

32. Most normal newborns are thought to have sufficient stores of iron. However, iron deficiency is still a
common problem during infancy. One of the following statement is true regarding iron deficiency in
infants:
A. Human milk contains less iron than most formulas, thus breastfed infants are prone to
develop anemia than formula fed infants.
B. Amount of iron stores at birth and its absorption are variable, thus onset of iron deficiency
may also vary.
C. Inadequate dietary intake of iron is the most common cause of this problem.
D. Rapid destruction of RBC during infancy

Ans: B
Although human milk contains less iron than most formulas iron is absorbed 2-3x more efficiently
from human milk than from cow’s milk. Iron deficiency secondary to inadequate dietary intake is
not common before 6 months of life.

33. An otherwise healthy 6 month old infant was brought to your clinic because of restlessness, crying
and failure to gain weight. What is the possible cause of the child’s problem:
A. This infant is suffering from septicemia
B. Late introduction of complementary feeding
C. Possibility of an abnormal mother-infant bonding
D. All of the above

Ans: D
Underfeeding is suggested by restlessness, crying, slow weight gain or actual weight l oss. All
of the above can cause infant’s failure to take sufficient quantity of food.

34. The most common cause of death from physical abuse is:
A. Intra-abdominal injuries
B. Burns
C. Intentional Head Trauma (IHT)
D. Poisoning

Ans: C
More than 95% of serious intracranial injuries during the 1st year of life are the result of IHT.
Intra-abdominal injuries from impacts are the second most common cause of death in battered
children.

35. Bruises are the most common manifestations of child abuse. Appropriate age of bruises allows
physicians to correlate history with actual age of injury. A bruise that is characterized as purple or
bluish is approximately:
A. 0-48 hours

24
B. 48-72 hours
C. 4-7 days
D. > 7 days

Ans: B
Bruise that is fresh (0-48 hours) is red; purple-blue is 48-72 hours; yellow-green 4-7 days; brown > 7
days.

36. A 2 year old male child was seen at the ER due to 2nd degree burns of both hands. What would you
do?
A. Admit the patient since you cannot clearly tell if the burns are intentional or not
B. Treat the burns and send home the patient
C. Refer to the surgeon on duty for management of the burns
D. Ask for the immunization status of the patient

Ans: A
In cases where the diagnosis is unclear, always admit the patient. The parents should be told why
an inflicted injury is suspected.

37. The use of helmets, seat belts, knee and elbow pads are interventions that:
A. prevent the occurrence of the injury-producing agent
B. will attempt to reduce the likelihood of injury by modifying the transfer of energy to the victim
C. will limit the impact of injuries on the victims
D. all of the above
Ans: B
The use of seat belts and other safety devices will modify the rate of release of the hazard from
its source.

38. A reliable index of long-term glycemic control is provided by measurement of :


A. FBS
B. Urine sugar
C. Glycosylated hemoglobin
D. Oral glucose tolerance test
Ans: C
39. The first and rate – limiting step in steroid synthesis is the conversion of cholesterol to: Aldosterone
A. Cortisol
B. Pregnenolone
C. sex steroids
Ans: C

40. At what age is onset of puberty considered precocious in girls?


A. < 7 years of age
B. < 8 years of age
C. < 9 years of age
D. < 10 years of age

Ans: B

41. A newborn with congenital Hypothyroidism is diagnosed and treated adequately by two weeks of age.
It can be anticipated that with continued treatment he will have:
A. Short stature but normal intelligence
B. Short stature and slight mental retardation
C. Normal growth and development
D. Normal stature but a mild degree of mental retardation

Ans: C

42. A 1 year and 6 months old child has congenital adrenal hyperplasia. He carries a boy’s name, and
had been raised as a boy. P.E shows a moderately enlarged phallus but empty scrotal sacs.
Chromosome analysis shows an XX pattern. If you were the doctor, you would:
A. continue to raise him as a boy
B. ask parents how they feel and follow their wishes
C. advise the parents that it would be best to raise the child as a girl after surgical correction of
the external genitalia
D. wait until the child is old enough to decide which sex to assume

Ans: C

25
43. The most severe form of mucopolysaccharidoses due to the deficiency of alpha-L- iduronidase is:
A. Hurler’s syndrome
B. Hunter’s syndrome
C. Morquio’s syndrome
D. Sanfillippo syndrome
Ans: A

44. Inborn error of amino acid metabolism associated with mousy or musty odor of the urine is:
A. Tyrosinemia
B. Hawkinsinuria
C. Phenylketonuria
D. Oast house urine disease
Ans: C

45. A 2 week old baby boy, delivered NSD, TERM, AGA, was admitted due to poor suck and vomiting,
upon admission patient was stuporous, dehydrated with sweet smelling urine. The most likely
diagnosis:
A. Penylketenuria
B. Oast house urine disease
C. Maple syrup urine disease
D. Xanthunuria

Ans: C

46. Lipid Storage disease associated with the deficiency of the lysosomal enzyme B- hexosaminidase A
is:
A. Gaucher Disease
B. Lesch-Nyhan Disease
C. Niemann-Pick Disease
D. Tay-Sachs Disease

Ans: D

47. Jesus, a 7 year old 2nd grader was brought by her mother to their physician because he was always in
trouble at school and impossible at home. He cannot sit still at the dinner table or anywhere else,
except perhaps in front of the television. Physical examination revealed nothing more than a slight
difficulty in performing skillful motor acts and some clumsiness. The most likely diagnosis is:
A. Temporal lobe epilepsy
B. Conduct disorder
C. Attention deficit hyperactivity disease
D. Attention deficit hyperactivity disorder

Ans: D

48. An 18 year old farm worker was brought to the ER with symptoms of abdominal cramps, excessive
salivation, vomiting diarrhea and muscle fasciculation. History revealed that he has been applying
pesticides on a large area of the banana plantation where he worked. The most common cause of his
problem:
A. Dioxin poisoning
B. Organophosphate poisoning
C. Hydrocarbon poisoning
D. Heavy metal poisoning

Ans: B

49. A 20 year old G1P0 woman gave birth to a baby boy with Down Syndrome. Her first pregnancy
resulted in abortion. Such chromosomal abnormality is usually due to:
A. Translocation
B. Nondisjunction
C. Mosaicism
D. Point mutation

Ans: A

50. A 12 year old female was brought in for consultation because of poor performance in school
especially in mathematics. On physical examination she was found to have short stature, low

26
posterior hairline, webbed neck, widely spaced nipples and sexual infantilism. The most likely
diagnosis:
A. Klinefelter Syndrome
B. Turner Syndrome
C. Edward Syndrome
D. Adrenogenital Syndrome

Ans: B

51. Which of the following organisms is the major cause of severe systemic and focal infections in
newborns?
A. Staphylococcus aureus
B. Streptococcus viridans
C. Group B streptococcus
D. Pseudomonas species

Ans: C

52. A four year old boy was brought to your clinic because he was exposed to his grandmother who was
coughing out blood for two months already. He has good weight gain and appetite, and has no
chronic cough nor fever. You administer a Mantoux tuberculin test and the reading after 72 hours is
15mm. What category does this child belong to?
A. TB Exposure
B. TB Infection
C. TB Disease
D. TB Inactive

Ans: B

53. A 5 year old girl developed fever, coryza and conjunctivitis. After 5 days, still with fever, she was noted
to have maculo-papular rashes and cervical lymphadenopathy. What is your diagnosis?
A. Rubella
B. Roseola
C. Fifth’s Disease
D. Rubeola

Ans: D

54. A mother with a known herpes simplex type 2 infection gave birth vaginally to a full term healthy
looking baby boy with the assistance of a hilot. What is the next immediate step to do after
resuscitation?
A. Give herpes simplex vaccine intramuscularly within first 12 hours of life
B. Send baby to the hospital for admission and intravenous acyclovir treatment
C. Advise mother to closely observe baby for appearance of oral lesions
D. Start first dose of oral acyclovir and refer to a pediatrician for further management

Ans: B

55. People of all ages can develop acute rheumatic fever, but it usually occurs in children
A. 0 – 1 year old
B. 2 – 4 years old
C. 5 – 15 years old
D. 16 – 19 years old

Ans: C

56. The diagnosis of acute rheumatic fever is mainly determined by


A. blood culture
B. clinical signs
C. electrocardiogram
D. acute phase reactants

Ans: B

57. NOT TRUE regarding mitral stenosis:


A. It is usually rheumatic in origin seen in older children and adolescents
B. It may cause concentric hypertrophy of the LV

27
C. The heart murmur is diastolic in timing
D. It can be managed with balloon valvuloplasty

Ans: B

58. A 7 year old girl presents with a tender and swollen right knee as well as a more recently appearing
swollen left ankle. She also has fever. Which of the following modified Jones criteria does the patient
fulfill?
A. 1 Major 1 minor
B. B.1 Major 2 minors
C. C.2 Majors
D. D. 2 Minors

Ans: A

59. The patient is 10 year old who had frequent tonsillitis and he had complained of migratory joint
swelling, intermittent high grade fever, palpitations and easy fatigue. PE includes BP 180/20 CR
110/min. Heart dynamic precordium, AB at 6 th LICS, AAL, S1 normal, S2 split P2 accentuated, Grade
3/6 diastolic blowing murmur LUSB radiating to apex, Peripheral pulses bounding. The diagnosis is
that he has RHD. What is the most likely cardiac pathology?
A. mitral insufficiency
B. mitral stenosis
C. aortic insufficiency
D. aortic stenosis

Ans: C

60. What is the most common cause of asthma attacks in the infancy period?
A. Viral infection
B. Stressful activity
C. Exposure to allergen
D. Food hypersensitivity

Ans: A

61. During the first year of life, the most common scenario in a child with adverse reaction to foods is:
A. Rashes after intake of soy-based foods
B. Drowsiness after intake of cured meat
C. Pruritic erythematous patches after intake of cheese
D. Watery, blood-streaked stools after cow’s milk intake

Ans: D

62. A preschooler was brought to the ER for difficulty of breathing. She was noted to be breathless while
talking in phrases and prefers to sit when examined. On PE, her RR is >40/minute, wheezing was
heard throughout expiration, was tachycardic and PEFR was 55%. What is the classification of
severity of this child’s acute asthma attack?
A. Mild intermittent
B. Mild persistent
C. Moderate persistent
D. Severe persistent

Ans: C

63. A 7-year-old boy was brought to the ER due to tightness in the chest. Earlier, he mentioned that he
was stung by a bee. Around 30 minutes after the sting, he felt warm and had difficulty swallowing his
saliva. He started feel dizzy and was noted to have difficulty of breathing. On PE he was noted to
have a BP of 50 palpatory, HR of 50/min regular in rhythm, RR of 50/min, with wheezing all over his
lung fields. What is the immediate treatment of choice for this patient?
A. Oral corticosteroid
B. Inhaled beta-2 agonist
C. Intramuscular epinephrine
D. Oral antihistamine

Ans: C

28
64. Based on the National Prevalence Survey, how many mm induration response to PPD 5 TU test,
differentiate the TB infected from the non infected Filipino?
A. 5
B. 8
C. 10
D. 15
Ans: C

65. A 2 year old boy has recurrent breathing pauses and snoring during sleep, consulted at the ER due to
difficulty of breathing. On PE his tonsils are swollen and enlarged. Your most commonly diagnosis is?
A. Acute epiglottitis
B. Acute tracheitis
C. Obstructive sleep apnea
D. Diptheria
Ans: C

66. A 6 month old male infant has mild upper respiratory tract infection and low grade fever gradually
develops into respiratory distress characterized by wheezing and dyspnea. The most likely diagnosis
is?
A. Bronchial asthma
B. Acute bronchiolitis
C. Acute bronchitis
D. Laryngotracheobronchitis (LTB)
Ans: B

67. NOT included in the management of Acute Laryngotracheobronchitis:


A. Give O2 and supportive care
B. Start racemic epinephrine by aerosol
C. Add inhaled steroids
D. Admit and start cephalosporins
Ans: D

68. A 2 year old with class III childhood TB would have the following characteristics EXCEPT:
A. (+) history of exposure to an adult with active TB
B. (+) mantoux tuberculin test
C. abnormal chest radiographs suggestive of PTB
D. Started on INH, Rifampicin, PZA and Ethembutol
Ans: D

69. What is the main stimulus for red blood cell production?
A. Anoxia
B. Infection
C. Hemorrhage
D. Immunodeficiency
Ans: A

70. A 4 year old child with brain tumor underwent radiotherapy and chemotherapy. After 2 years, the child
was noted to be stunted (height at p35). What is the likely reason for such a finding?
A. Decrease in brain size due to radiotherapy lessened capacity of growth hormones to
induce growth
B. Damage to hypothalamic axis has decreased amount of growth hormones causing
stunting
C. Chemotherapy caused hypoperfusion and ischemia of the brain
D. Radiation induced hypoplasia of the skull which limited brain development
Ans: B

71. Laboratory test to confirm the diagnosis of neuroblastoma:


A. Urine vanillylmandelic acid (VMA)
B. Urine catecholamins
C. Serum alpha-fetoproteins (AFP)
D. Serum beta- human chorionic gonadotrophin (B-HCG)

Ans: A

72. A 2 year old child was brought to the emergency room for on and off fever and easy bruisability for 4
weeks already. On PE, you noted palmar pallor, generalized lympadenopathy and hepatomegaly.
What is the most likely cause for his signs & symptoms?

29
A. Connective tissue disease
B. Chronic infection
C. Blood malignancy
D. Nutritional anemia

Ans: C

73. A 5-year-old boy was brought to the OPD for frequent vomiting and abdominal distention. He has had
poor appetite for almost one month and would complain of constipation and reddish urine. On PE,
BP= 130/90, T= 38.2C. You palpated a mass on the right side of the abdomen which was firm, not
movable and non-tender. What is your diagnosis?
A. Rhabdomyosarcoma
B. Hirschsprung’s disease
C. Impacted feces
D. Wilms’ tumor

Ans: D

74. The most common cause of bacterial meningitis at 2 months of age is:
A. Group B Streptococcus
B. Mycoplasma pneumoniae
C. Psuedomonas aeruginosa
D. Listeria monocytogenes

Ans: A

75. Lumbar puncture is indicated in the following conditions:


A. Child with encephalitis with signs of increased intracranial pressure
B. Child who present with headache, associated with left-sided hemiparesis, facial asymmetry
and ptosis
C. Child who presents with fever, generalized seizures and nuchal rigidity
D. Child with fever, seizures, purpuric rash and hematoma at all puncture sites

Ans: C

76. A 10-year-old boy developed severe headache, photophobia and vomiting. He is febrile. P=180/100
mm Hg. Results of cranial nerve and motor examinations are normal. He has nuchal rigidity and
extensor plantar response, (+) Babinski. The most appropriate to obtain this time is:
A. CSF analysis
B. Electroencephalogram
C. CT scan
D. Skull x-ray

Ans: C

77. You are evaluating a 5-year-old girl who has fever, vomiting, and nuchal rigidity. CSF examination
reveals WBC=1650, 85% segmenters, 15% lymphocytes, glucose=20 mg/dl and protein=250 mg/dl.
Gram stain showed Neisseria meningitides. The best choice of parenteral antibiotic for this patient is:
A. Penicillin
B. Ceftriaxone
C. Nafcillin
D. Vancomycin

Ans: B

78. A 7 year old child was admitted for Acute Post Strep Glomerulonephritis. What is the usual presenting
symptom of the patient
A. Headache
B. Dysuria
C. Periorbital edema
D. Abdominal Pain

Ans: C

79. Anemia in Post Strep Glomerulonephritis is due to:


A. High grade hemoglobin
B. Hematuria

30
C. Hemodilution
D. decrease production of RBC

Ans: C

80. Which of the following indicate that the patient is experiencing a severe complication of acute
glomerulonephritis?
A. Temperature of 38.8 °C
B. Blood Pressure of 140/92 /mm Hg
C. Severe Sodium of 140 mg/ L
D. Weight loss of 2 lbs

Ans: B

81. A 3 year old boy come in the ER with abdominal pain generalized edema, BP 90/60 mmHg T36.8C
HR85/min BUN 217mg/dl Cr).5 mg/dl, Urine Protein is1800mg/24h. Impression is:
A. Congestive Heart Failure
B. Idiopathic Nephrotic Syndrome
C. Systemic Lupus Erythematosus
D. Post-streptococcal Glomerulonephritis

Ans: B

82. The most common etiologic agent involved in acute hematogenous osteomyelitits in children is
A. Group B streptococcus
B. Group A streptococcus
C. Staphylococcus aureus
D. Hemophilus influenza type B

Ans: C

83. Which of the following statements is TRUE?


A. Septic arthritis is a disease most commonly found in adolescent males
B. In septic arthritis, the hips and knees are the most commonly affected joints
C. In a child with septic arthritis of the hip, redness swelling and warmth are often detectable on
PE
D. Children with transient synovitis never present with fever
Ans: B

84. Cardinal features of Ehlers-Danlos syndrome include all of the following EXCEPT:
A. hyperextensible doughy skin
B. palpable purpura
C. joint hypermobility
D. vascular fragility and brusing
Ans: B

85. These are group of bone diseases in which the ossification centers undergo avascular necrosis
followed by resorption, fragmentation of dead bones and finally regeneration and replacement of
reparative bone tissues.
A. Osteochondroma
B. Osteochondroses
C. Osteopetrosis
D. Osteochondritis
Ans: B

86. The most feared complication of Kawasaki Disease:


A. Pericarditis
B. Coronary Aneurysm
C. Aseptic Meningitis
D. Myocarditis
Ans: B

87. A child who has low grade fever, abdominal pain, arthritis, microscopic hematuria and purpuric rash
only in the lower extremity. He most likely has
A. Meningococcemia
B. Poststreptococcal Glomerulonephritis
C. Henoch-Schoenlein purpura

31
D. Wegener’s granulomatosis

Ans: C

88. In the diagnosis of Marfan syndrome, the most useful information is


A. History of consanguinity
B. Chromosomal mapping to detect deletions of Chromosome 15
C. Width of aortic root exceeding more than 95th percentile
D. Measurement of fibrillin precursors in peripheral leucocytes
Ans: C

89. A 13 year old girl is seen because of 4 weeks history of body malaise, anorexia, intermittent fever and
arthritis of ankles, wrist and knees. She developed shortness of breath and dyspnea on exertion. PE
revealed BP 92/58, RR of 24, Pulse 125, crackleson both lung bases, muffled heart sounds
accompanied by friction rub. 2 D Echocardiography documents pericardial effusion. The most useful
screening laboratory test for this patient is
A. Serum Ig levels
B. Antinuclear antibody
C. Serum complement levels
D. Human lymphocyte antigen (HLA)

Ans: C

90. A 14 year old male is referred for evaluation of heart murmur noted on sports pre-participation
physicals. PE reveals a very tall thin boy who has pectus deformity of the chest, hyperextensible
joints, and apical holosystolic murmur with mid diastolic rumble. The most helpful diagnostic
evaluation to determine the cause of these findings is
A. Skeletal survey
B. Slit lamp eye examination
C. Measurement of plasma amino acids
D. MRI of the spine and sternum

Ans: B

91. Which of the following drugs is contraindicated for use by the breastfeeding mother?
A. Ergotamine
B. Carbamazepine
C. Phenytoin
D. Valproic acid

Ans: A

92. A 2500 gram infant who is born at 36 weeks AOG has a head circumference of 27 cm and crown-heel
length of 40 cm. Other findings include upturned nose, hypotonia, hypoplastic philtrum. The most
likely prenatal agent that would explain these findings is
A. Alcohol
B. Cocaine
C. Marijuana
D. Opiates

Ans: A

93. Physical examination of a newborn female infant reveals meningomyelocoele, cleft lip and craniofacial
anomalies. The most likely prenatal experience to explain these findings is
A. alcohol
B. Lithium
C. Thiazides
D. Valproic acid

Ans: B

94. A 5 year old boy who is taking Carbamazepine for control of generalized seizure presents with signs
and symptoms of pneumonia. Which of the following antibiotics most likely cause toxicity if
prescribed concomitantly with Carbamazepine?
A. Ampicillin
B. Erythromycin
C. Vancomycin

32
D. Trimethoprim-Sulfamethoxazole

Ans: B

95. A 15 year old patient has asthma and is taking salbutamol or albuterol via metered dose inhaler. You
are asked the known side effects of this drug by this patient. The most common side effect to tell is
A. Seizure
B. muscle cramps
C. somnolence
D. tachycardia

Ans: D

96. Normal plasma osmolality is approximately


A. 265-275 mOsm/kg H20
B. 285-295 mOsm/kg H20
C. 305-315 mOsm/kg H20
D. 325-335 mOsm/kg H20

Ans: B

97. Hyponatremia defined as <130 mEq/L of sodium may be caused by the following EXCEPT:
A. Mineralocorticoid deficiency
B. Osmotic diuresis
C. Symptom of inappropriate ADH secretion (SIADH)
D. Diabetes insipidus

Ans: D

98. The following are consequences of hypokalemia EXCEPT:


A. Paralytic ileus
B. Prolonged QT interval
C. Increased neuromuscular excitability
D. Weakness

Ans: C

99. A 3 year old child with diarrhea who presented with lethargy, rapid feeble pulses and very sunken
eyeballs has an estimated fluid deficit of:
A. 30-50 ml/kg
B. 60-90 m/kg
C. ≥100 ml/kg
D. ≥150 ml/kg

Ans: C

100. An arterial blood gas determination taken on room air showing the following values pH= 7.3,
pCO2= 35 mmHg, HCO3= 16 mmol/L, PO2= 60 mmHg is indicative of:
A. Metabolic acidosis and normal oxygenation
B. Metabolic acidosis and hypoxemia
C. Respiratory acidosis and hypoxemia
D. Normal acid-base balance with hypoxemia

Ans: B

PEDIATRICS
Case I. Nos. 1-2

A 3 year old normal child was admitted to a hospital because of simple febrile convulsions
p. 1496
1. Routine management of the case includes:
A. careful search for cause of fever (0.5)
B. short term anti-convulsant prophylaxis (0.5)
C. Phenobarbital maintenance to prevent recurrence
D. Lumbar puncture

33
2. The anti-convulsant that may be effective in the acute management of prolonged febrile seizures is:
p1496
A. rectal Diazepam (0.33)
B. rectal Phenobarbital (0.33)
C. Intravenous Phenobarbital (0.33)
D. Intravenous Phenytoin

Case II. Nos. 3-4

A two-week old baby was noted to have a rapidly increasing head circumference. At birth, head
circumference was 40 cms, against a chest circumference of 34 cms. Present physical examination
showed a head circumference of 45 cms, wide, bulging anterior fontanel, gaping sutures, dilated scalp
veins and a (+) setting sun sign.

3. If the occiput is prominent, the primary consideration is: p 1489


A. Aqueductal stenosis (0.33)
B. Chiari malformation (0.33)
C. Dandy-Walker malformation (0.33)
D. Hydranencephaly
4. This drug will reduce the rate of CSF production: p1490
A. Dexamethasone
B. Acetazolamide (1.0)
C. Prednisone
D. Mannitol

Case III. Nos. 5 – 9


A 9 year old child was brought to the OPD clinic because of deterioration in school preformance, frank
dementia, myoclonic jerks and cerebellar ataxia. He had no immunization and contracted measles at 10
months of age, varicella at 2 years and mumps at 5 years

5. The most likely diagnosis is: p 844


A. progressive rubella panencephalitits
B. subacute sclerosing panencephalitis (1.0)
C. post-varicella encephalitis
D. adrenoleukodystrophy
6. The seizures are best controlled by: p 1498
A. Carbamazepine (0.5)
B. Valproate (0.5)
C. Phenytoin
D. Lamotrigine

Administration of this drug may prolong the child’s survival: p 844


A. Methisoprinol
B. Inosiplex (0.5)
C. Taurine
D. Intravenous immunoglobulin (0.5)

The most important hormone regulating renal calcium excretion is: p 200
A. Dihydroxy vitamin D (0.33)
B. Calcitonin (0.33)
C. Parathyroid hormone (0.33)
D. Thyrotoxin

A 2 year old female was noted to be oliguric for 24 hours. She was brought to the ER with Creatinine
levels of 350 mmol/l. Serum potassium level was 6.5 mmol/l. What possible maneuver can induce a
negative potassium balance and decrease serum potassium levels?
P 223
A. sodium bicarbonate infusion (0.25)
B. insulin/glucose infusion (0.25)
C. calcium gluconate (0.25)
D. kayexalate (0.25)

10. A 1 year old baby boy was brought to the ER due to seizure. On PE, the baby was
seem to have flexed wrist, fingers extended, thumbs adducted over the palms and the feet extended
and adducted. What is the possible cause of the seizures? P 224
A. grand mal seizure (0.25)
B. meningitis (0.25)

34
C. benign febrile seizures (0.25)
D. hypocalcemia (0.25)

11.A 3 year old boy has been having diarrhea for 5 days already with stooling of 3 – 5 x per day,
voluminous, watery in character. On PE, he was noted to be in moderate dehydration, with serum
sodium lever of 160 mmol/l. Patient was hydrated with D5water, however, after 2 hours of hydration.
The patient went into a seizure. What is the possible cause of the seizure?
P 217
A. Patient developed subdural effusion due to the hyperosmolality
B. There is an excess movement of water into cerebral cells during rehydration causing
cerebral edema
C. The patient had late manifestation of hypernatremia (0.5)
D. Patient had meningitis (0.5)

12. A 2 year old male was seen in the ER due to fever of 5 days duration. This was
associated with vomiting and anorexia. There were no cough, colds associated. CBC
revealed leukocytosis with predominance of Segmenters. Urinalysis revealed TNTC pus cells
and 5-10/hpf RBC. Impression : UTI. The gold standard for the diagnosis of UTI is:
P 1786-1787
A. Urine Culture and sensitivity (1.0)
B. Leukocyte esterase test
C. Urinalysis
D. Nitrite test

13. A 6 year old male was seen in the ER due to Tea colored urine of 2 days duration. This was
associated with periorbital edema, abdominal distention and grade 2 pitting edema of the lower
extremities.
Impression : Acute Glomerulonephritis/PSGN
Post streptococcal glomerulonephritis is most common in children aged: P 1740
A. 5 - 10years old (0.5)
B. 2 - 4 years old (0.5)
C. 13-18 years old
D. 1-2 years old

14. The best single antibody titer to document cutaneous streptococcal infection in PSGN
is: P1741
A. Deoxyribonuclease B antigen (DNase)
B. ASO titer (1.0)
C. Streptozyme test
D. Phadebact test

A term neonate with an uncomplicated birth history was noted to be cyanotic during the first few days of
life. On examination, there was central cyanosis and absence of heart murmur. Chest x-ray showed
normal heart size and diminished pulmonary vascular markings. Most likely diagnosis is
p 1529 - 1530
A. Pulmonary Valve Atresia (0.5)
B. Tetralogy of Fallot
C. Truncus Arteriosus, type I
D. Transposition of Great Arteries with VSD (0.5)

A 3 month old boy was noted to be cyanotic on crying when he was 2 months old. This became
persistent and more pronounced on exertion. On physical examination, the baby was cyanotic with a
grade 2 – 3/6 systolic ejection murmur at the 3rd – 4th ICSLPSB. This baby is suffering from
p 1525
A. Pulmonary Valve Atresia
B. Tetralogy of Fallot
C. Transposition of Great Arteries, no VSD (0.5)
D. Tricuspid Valve Atresia (0.5)

A 4month old baby girl was brought for consultation because of frequent cough and
colds accompanied by inability to consume her milk formula, fast breathing during feeding and chest
retractions. A murmur was heard accompanied by bounding peripheral pulses and wide pulse
pressure. The most likely diagnosis is p 1511
A. Atrial Septal Defect
B. Ventricular Septal Defect
C. Patent Ductus Arteriosus (1.0)
D. Pulmonic Stenosis

35
A 2 year old boy presented with a history of 5 days fever accompanied by irritability, bilateral conjunctival
injection, unilateral cervical lymphadenopathy, rashes in the diaper area and congested buccal
mucosa. In order to prevent complication, this boy should be given high dose aspirin and
p 825
A. Digoxin
B. Penicillin (0.5)
C. Diuretics
D. Intravenous immunoglobulin (0.5)

A 6 year old child was diagnosed as a case of acute rheumatic fever and received ten days course of
aqueous penicillin. In order to prevent the recurrence of group A streptococcal infection, this child
should receive p 876 - 879
A. Benzathine benzylpenicillin 1.2 million units intramuscular once
B. Benzathine benzylpenicillin 1.2 million units intramuscular every 21-28 days (1.0)
C. Oral penicillin 250 mg twice a day for 10 days
D. Oral erythromycin 250 mg twice a day for 10 days

A 10 year old boy who presents with difficulty in breathing accompanied by a displaced PMI to the left,
apical and systolic thrill and grade 4/6 holosystolic murmur at the apex, is most likely suffering from
an insufficient p 1570
A. Aortic valve
B. Mitral valve (1.0)
C. Pulmonic valve
D. Tricuspid valve

Children with small Ventricular Septal Defect are at risk for this complication
A. Hypoxic spells (0.5) p 1509
B. Heart failure
C. Pulmonary Artery Hypertension (0.5)
D. Infective Endocarditis

Case 1 Nos. 22 – 24

Lito, a 5 year old child, was seeking enrollment in a nursery school. Both the teacher and the
school physician evaluated him for growth and development. Birth history revealed that Lito was born
prematurely at 34 weeks with a weight of 1924 grams and a length of 42 cms. Head circumference was
31 cms. The expected anthropometric measurements includes:

22. Weight that has increased at least _________ from birth


A. 4x
B. 6x (0.5)
C. 8x (0.5)
D. 10x
23. An increased in length of
A. 22 inches (0.25)
B. 24 inches
C. 26 inches
D. 28 inches
24. Ideal head circumference for age is
A. 40 cms
B. 42 cms
C. 44 cms (1.0)
D. 46 cms

Case 2. Nos. 25 - 29

Buboy is a 7 month old infant brought tot the clinic for a well baby visit. His grandmother recalled
that his birthweight was 6 lbs but was not aware of the birth length and head circumference.
25. Buboy’s ideal length is
A. 24 inches (0.25)
B. 27 inches (0.25)
C. 31 inches (0.25)
D. 35 inches (0.25)
26.The expected increase in his head circumference is:
A. 2 cms (1.0)
B. 4 cms
C. 6 cms

36
D. 8 cms
27.Buboy’s ideal weight at 6 months is
A. 11 lbs
B. 13 lbs (1.0)
C. 15 lbs
D. 17 lbs
28.When Buboy’s is offered an object he is expected to
A. use his index finger to get it
B. get the object and drop it (0.5)
C. cast the object
D. transfer the object form hand to hand (0.5)
29.The motor milestone expected for age is
A. sitting propped up on hands
B. crawling (0.5)
C. Pulling up to stand (0.5)
D. Cruising

30. A 9 year old boy is suffering from greasy foul watery stool of 2 weeks duration accompanied with
abdominal cramps and abdominal distention. You are entertaining the possibility of Giardiasis but the
stool examinations done thrice were negative. The next best procedure to do is :
p 1125 - 1127
A. abdominal ultrasound
B. entero – test or string test (1.0)
C. duodenal biopsy
D. polymerase chain reaction

31. A 2 year old girl was brought in for consultation because of watery stool and
vomiting of 3 days duration. Modified acid fast staining of the stool revealed a 2-
6 micrometer red oocysts . The most likely diagnosis in this case is : p 1128
A. giardiasis
B. balantidiasis (0.5)
C. cryptosporidiosis (0.5)
D. amebiasis

32. A 4 year old girl with protuberant abdomen has recurrent colicky periumbilical
pain. Fecalysis shows a broadly ovoidal ova with thick shell and outer
mammilated covering about 40-60 micrometer. Because of high rate of
reinfection, chemotheraphy has to be repeated at : p 1155-56
A. 1 month interval (0.25)
B. 2 months interval (0.25)
C. 3-6 months interval (0.25)
D. 8-12 months interval (0.25)

33.An 18 year old male from Samar presented with abdominal distention ,
hepatomegaly nad some signs of portal hypertension. Fecalysis revealed small egg
with short curved spine. The specific drug for this condition is : p 1167
A. diethylcarbamazine (0.33)
B. albendazole (0.33)
C. ivermectin (0.33)
D. praziquantel

34. A 6 month old infant with feeding problem was noted to have diffuse papular lesions , chorioretinitis on
the right eye and enlarged head circumference . IGM-SAGA test was positive. This patient can best
be treated with . p 1164 - 1166
A. pyrimethamine + sulfadiazine + calcium leukoverin (0.25)
B. Metronidazole (0.25)
C. Iodoquinol (0.25)
D. Chloroquine phosphate (0.25)

35. 3 year old boy who owns a puppy has a history of pica. He develops fever, cough with wheezing and
hepatomegaly. The test that that will most likely helps in making an accurate diagnosis is :
p 1164 - 1166
A. stool examination
B. ELISA (0.5)
C. COPT (0.5)
D. PCR

37
36. A 24 month old infant with a normal weight came in for diarrhea with severe dehydration. You opted
to administer plain LRS. The total amount of fluid that should be administered in 3 hours is
approximately
A. 800 ml
B. 1000 ml
(0.5)
C. 1200 ml
(0.5)
D. 1400 ml

37. A 12 month old infant came in for bloody stool, high fever, severe abdominal pain and toxicity for the
past 32 hours. Stool exam revealed 60 fecal leukocytes, plenty of bacteria and amoebic cysts. Most
likely the cause of the diarrhea is
A. Shigella (0.33)
B. Amoeba (Entamoeba histolytica) (0.33)
C. Salmonella
D. Enteroinvasive Escherichiae coli (0.33)

38. A 3 week old infant came in for non-bilous vomiting occurring immediately after feeding. This was
followed by progressive loss of fluids and electrolytes and dehydration. Feeding was alright. Most
likely you are dealing with
A. Duodenal atresia
B. Esophageal atresia (0.5)
C. Hypertrophic pyloric stenosis (0.5)
D. Duodenal stenosis

39. A fairly nourished one year old in some dehydration will be needing approximately this
amount of rehydration fluid during the initial replacement therapy
A. 200 – 400 ml
(0.33)
B. 400 – 600 ml
(0.33)
C. 600 – 800 ml
(0.33)
D. 800 – 1000 ml

Case 1. No. 40

A 2 year old child was brought to your clinic with a history of having fallen down the stairs from a height of
6 steps. You would suspect child abuse because of the presence of the following:

40. A. bruises on the back


B. 2 cm skull fracture – parietal area (0.5)
C. clavicular fracture – medial aspect (0.5)
D. contusion/abrasions on the upper arm and legs

REFERENCE: APMC Workbook on Child Protection

A 2 year old girl was brought to the ER with a history of paracetamol ingestion. Mother claimed that 3
hours PTC, the child ingested an almost full 30 ml bottle of Paracetamol with a concentration of 250
mg./kg. the child weighed 10 kg.

41. This statement is TRUE


A. She has ingested a toxic dose of paracetamol (0.5)
B. Children have a lower incidence of toxic plasma levels after ingestion than do adolescents
(0.5)
C. Ipecac should be given as an initial GI decontaminant
D. Activated charcoal should not be given if NAC is to be administered

42. Initial symptom include: P 2366 - 2367


A. oliguria (0.25)
B. jaundice (0.25)
C. diaphoresis (0.25)
D. abdominal pain (0.25)

A 10 year old child, diagnosed to have Juvenile Rheumatoid Arthritis and maintained on salicylate was
brought to the ER with fever, headache and body malaise. You considered the possibility of salicylate
toxicity

38
43. This statement is TRUE: P 2367 - 2368
A. Serum salicylate levels should be taken 6 hours after the last salicylate ingestion
B. Chronic toxicity usually presents with metabolic acidosis (1.0)
C. A salicylate level of 10 – 20 mg/dl will rule out chronic toxicity
D. Gastric decontamination should be performed immediately
44. The management of salicylate toxicity would include the administration of
A. Potassium (0.5)
B. Calcium (0.5)
C. Acetazolamide
D. Mannitol p 2367 - 2368

45. A major criteria in the diagnosis of atopic dermatitis is P 682


A. chronic relapsing course (0.25)
B. elevated serum IgE (0.25)
C. early age of onset (0.25)
D. white dermatographism (0.25)

46. A 5 year old with asthma who have daytime symptoms of >2 times a week, nocturnal symptoms of
> 2 times monthly. PEFR > or equal to 809. PEFR variability 20 – 30% is diagnosed to have
P 673
A. intermittent asthma
B. mild persistent asthma (0.33)
C. moderate persistent asthma (0.33)
D. severe persistent asthma (0.33)

47. For controller/maintenance treatment, this patient may need EXCEPT P 675
A. inhaled corticosteroid (0.33)
B. cromolyn (0.33)
C. nedocromil
D. oral steroids (0.33)

48. A child who suddenly develops difficulty of breathing, urticarial rashes, abdominal pain 5 minutes after
eating crabs need immediate administration of
A. intramuscular antihistamine (0.33)
B. intravenous antihistamine (0.33)
C. intramuscular corticosteroid
D. epinephrine 1:1000 intramuscular (0.33) P 687

49. The only effective treatment of B-cell disorder


A. Thymic cell transplant
B. Bone marrow transplant (0.5)
C. Regular administration of gammaglobulin (0.5)
D. Plasma transfusion P 598

Case 1. Nos. 50 - 51
Baby girl OA was born to a 24 year old primigravid at 39 weeks AOG, NSD after an
uncomplicated pregnancy. She weighed 2850 grams, AS 9 & 10, On the 12 th hour of life, she was noted
to be jittery. There was fair suck. Reflexes were good. Serum calcium was 4 mg/dl. HGT was 70 mg%

50. The most likely diagnosis is


A. Transient idiopathic neonatal hypocalcemia (0.5)
B. Early neonatal hypocalcemia (0.5)
C. Small for age
D. Infant of diabetic mother, hypocalcemia secondary
51. The most probable cause of her disorder is
A. metabolic imbalance of hypoglycemia (0.25)
B. presence of cytoplasmic antibodies (0.25)
C. functional immaturity of PTG (0.25)
D. prematurity

Case 2. Nos. 52 - 53

A 3 month old male was admitted due to frequent vomiting since 2 weeks of age. He had poor suck,
weighed 3.5 kgs, dehydrated, hypotensive

52. Differential diagnoses includes all of the following EXCEPT


A. septic shock (0.33)

39
B. adrenocortical insufficiency, etiology to be determined
C. failure to thrive (0.33)
D. Acute gastroenteritis (0.33)

53. Management should include


A. DAT, Prednisone, NaCl supplementation
B. NPO, D5NSS, hydrocortisone succinate, fludrocortisone (0.33)
C. NPO, D5NSS, hydrocortisone succinate (0.33)
D. NPO, D5NSS, fludrocortisone and NaCl supplementation (0.33)

Case 3. No. 54
A 9 year old male had a weigh of 120 lbs, height of 4 feet complains of paroxysmal attacks of
headache and nape pain. The dark pigmentation on the neck, axilla, nape and his acne bothers him a lot.
On consultation, his BP was 120/80 mmHg, CR: 90/min

54. The patient has


A. normal weight, height and BP
B. increased weight, normal height and increased BP (0.33)
C. normal weight, increased height and increased BP (0.33)
D. increased weight, increased height and increased BP (0.33)

Case 1. Nos. 55 – 56

A diabetic 33 year old Gravida 1 Para 1 type O+ mother delivered 5 weeks earlier than her
expected date of confinement. The baby was delivered in a lying-in and was noted to be limp, cyanotic
face and pale body, gasping, pulses weak and no response on suctioning

55. If your are present on the delivery of this baby. The Apgar score at 1 minute is
A. 0
B. 1 (0.5)
C. 2
D. 3 (0.5)
56. If he was fed on expressed breast milk and no vitamin K given at birth, the anticipatory
condition that could probably set in
A. Hemorrhage (0.33)
B. Jaundice (0.33)
C. Diarrhea (0.33)
D. Necrotizing enterocolitis

57. A 20 year old primigravid mother now on her 31st week age of gestation consulted for painless vaginal
bleeding. Stat pelvic ultrasound showed placenta previa totalis and thus emergency caesarean
section was done. What immediate problem do you expect in the baby?
A. Meconium Aspiration Syndrome
B. Respiratory Distress Type I (0.33)
C. Apnea of Prematurity (0.33)
D. Respiratory Distress Type II (0.33)
58. The pulmonary problems brought about by Respiratory Distress Type I are due to
A. low compliance (0.33)
B. high resistance (0.33)
C. high functional residual capacity
D. low dead space (0.33)

59. A post term was delivered thru caesarean section because of non-reassuring fetal heart rate status.
Baby was born through thickly meconium stained amniotic fluid, weighed 2400 grams and had poor
apgar score. He is at risk for the following problem:
A. Respiratory Distress Type II (0.5)
B. Persistent pulmonary hypertension (0.5)
C. Meconium plug
D. Meconium ileus

60. One of the following is responsible for decrease pulmonary venous return at birth
A. increase paO2 (0.25)
B. increase paCO2 (0.25)
C. decrease pH (0.25)
D. all of the above (0.25)

61. Unconjugated hyperbilirubinemia in an otherwise normal 8 day old infant can be attributed to
A. breastmilk jaundice (0.33)

40
B. breastfeeding jaundice (0.33)
C. sepsis neonatorum
D. galactosemia (0.33)

62. A 3 year old child with malnutrition is being evaluated for admission at the ward. The anthropometric
index that reflects recent nutritional experience of the child is p170
A. weight for height (0.5)
B. body mass index (0.5)
C. mid-arm muscle circumference
D. triceps skin fold

63. A 1 ½ year old male consulted the OPD for persistent diarrhea of 2 ½ weeks. Physical examination
showed he had dry, scaly eczematous dermatosis at the perioral, cheeks, perianal and acral areas
with growth retardation, alopecia and reddish tint of hair. He is likely to have a deficiency of
p 2248
A. Iodine (0.25)
B. Magnesium (0.25)
C. zinc (0.25)
D. fluoride

64. A mother consulted the clinic to manage her 9 year old obese child. Her condition is most often
associated with the following features EXCEPT p 176
A. hyperpigmentation in the neck and skin creases
B. tall stature (0.5)
C. slightly advanced bone age (0.5)
D. delayed onset of puberty

65. A child with non-edematous protein energy malnutrition is now on the last phase of therapy. Iron
therapy for this child is not given in the early phase of treatment because p 173
A. free iron in the early phase may exacerbate oxidant damage (0.25)
B. iron does not promote growth in the early phase of treatment (0.25)
C. iron has no significant effect on the host’s defense mechanism (0.25)
D. the high levels of transferring is still enough to facilitate absorption (0.25)

66. A newborn was delivered to a mother who is positive for the hepatitis B virus. The following is/are
TRUE regarding breastfeeding of the mother in this infant p 159
A. may breastfeed only once maternal infection is fully cured (0.5)
B. should stop breastfeeding because hepatitis B is a highly contagious disease
C. active and passive immunizations permit breastfeeding with little risk to the infant
(0.5)
D. breast feeding is the major route of transfer of infection to the body

67. An eleven (11) year old boy will most likely p 51 - 53


A. groom in favor of the peer group “uniform” (0.5)
B. be able to engage in intimate and empathetic relationship with another person
C. be idealistic (0.5)
D. none of the above

68. An adolescent boy was seen at the clinic for a routine check up. On history and physical examination,
he was noted to be very self-conscious and his peers were all boys. Genital examination showed
scanty long, slightly pigmented pubic hair at the base of the penis and scrotum was slightly enlarged.
Based on the above history and PE one of the following is also expected
p 51
A. voice change is already completed
B. presence of hair over the chest
C. peak of penile growth (0.5)
D. ejaculation usually in response to masturbation (0.5)

69. In the HEADS FIRST acronym for psychosocial history of adolescence, “R” stands for
A. Rebelliousness
B. Recreation (1.0)

C. Responsibilities
D. All of the above

70. The first pubertal sign in boys is p 54 - 56


A. appearance of mustache
B. broadening of the shoulder (0.33)

41
C. increase muscle mass (0.33)
D. testicular enlargement

71. A 6 year old female was seen at the ER because of fever of 4 days. This was associated with sore
throat, decrease in appetite and slight dysphagia. Physical examination revealed hyperemic throat with
petechiae on the soft palate. Complete blood count revealed WBC 14,000 with Segmenters of 0.78.
Your most likely diagnosis is p 1393
A. Bacterial pharyngitis (0.5)
B. Acute epiglottitis (0.5)
C. Right peritonsillar abscess
D. Acute laryngitis

72. The drug of choice for the above case is p 1393


A. Cloxacillin sodium
B. Penicillin (0.33)
C. Ampicillin (0.33)
D. Cephalexin (0.33)

73. A 9 month old male infant was brought to the ER because of DOB of 1 day. He had intermittent low
grade fever for 4 days with nasal discharge and non-productive cough. A day prior to consult, he had
fast breathing and decreased appetite. But he remained to be playful. Physical examination revealed
RR of 60’s/min, T: 38oC, intercostal and occasional wheezing with inferior displacement of a normal-
sized liver. The most likely etiology of the disease is
A. Parainfluenzae
B. Adenovirus (0.5)

C. Influenzae
D. RSV (0.5) p 1415

74. A 4 month old male infant was noted to have stridor during sleeping for the past 1 month. The stridor
disappears when the infant is carried in an upright position. There was no associated fever and patient
was apparently well since birth. The most likely diagnosis is
p 1409
A. Laryngitis
B. Laryngeal foreign body
C. Laryngomalacia (0.5)
D. Bronchomalacia (0.5)
75. Signs and symptoms of above condition usually resolve by p1409
A. 6 months
B. 12 months (0.5)

C. 18 months (0.5)
D. 24 months

76. A 3 year old healthy child was left alone in the room playing. When the mother went back after a few
minutes, she saw her child lying on the floor, awake, aphonic and apneic. She was rushed to the ER.
Your most likely diagnosis is complete upper airway obstruction due to foreign body aspiration. The
maneuver/s to do on the way to the hospital is/are P 1411
A. Heimlich (0.33)
B. back blows and chest thrusts (0.33)
C. blind finger sweep of the oral cavity (0.33)
D. A and C

77. An induration of > or = to 5 mm tuberculin skin test reaction is said to be positive if the following
condition/s is/are present in a child p 962
A. receiving corticosteroids of < or = to 10 mg per 24 hours for 2 weeks
B. clinical or radiographic findings of TB disease
C. esposure to laryngotracheobronchitis
D. all of the above

78. One of the following viral exanthems is difficult to recognize clinically as symptoms mimic
other viral infections as mild illness with lymphadenopathy, slight fever, concurrent with generalized
erythematous maculopapular rash starting on the face
A. Roseola infantum p 1033
B. Postnatal rubella (0.5)
C. Measles (0.5)
D. Dengue fever

42
79.A 2 month old infant was brought to the clinic for immunization. Physical examination findings were all
within normal. Which of the following vaccines may be given?
A. DPT (1.0) p 1177
B. Hepatitis A
C. Measles vaccine
D. Typhoid vaccine

80.A 3 year old girl was transfused with 3 units of fresh frozen plasma due to dengue hemorrhagic
fever. When is the soonest time that MMR can be given after the transfusion?
A. > 3 months after (0.33) p 423
B. > 6 months after (0.33)
C. > 9 months after (0.33)
D. > 12 months after

81.A 2 year old child was admitted under your service with a working diagnosis of measles to prevent
further complications vitamin A must be given at a dose of p 1029
A. 50,000 IU
B. 100,000 IU
C. 150,000 IU
D. 200,000 IU (1.0)

82. A 6 year old child and his family spent New Year in Baguio City. A high possibility of exposure to
meningococcemia prompted the mother to ask for possible chemoprophylaxis. The drug that can help
in such case is p 899
A. penicillin
B. rifampicin (1.0)
C. erythromycin
D. chloroquine

83. While doing a routine physical examination to a 4 months old infant you found out that his left testes is
undescended. As a preventive measure against the possibility of having cancer later in life, you advise
the mother to have her child undergo orchiopexy if the testes will not descend before reaching the age of
p 1817
A. 5 -8 months old
B. 9 – 15 months old (0.5)
C. 2 – 4 years old (0.5)
D. 5 – 6 years old

84. If paralysis occurs in a 1 year old infant infected with poliovirus when is it expected to be observed?
P 1037
A. 14 days after onset of the illness (0.33)
B. 1 day after onset of the illness (0.33)
C. 3 – 8 days after onset of the illness (0.33)
D. 30 days after onset of the illness

85.Vaccine associated paralytic poliomyelitis p 1039


A. occurs 7 – 14 days after giving inactivated polio virus vaccine
B. occurs 7 – 14 days after giving oral polio virus vaccine (0.5)
C. occurs 15 – 30 days after OPV (0.5)
D. occurs 15 – 30 days after IPV

43
86.Ninety (90%) of viral meningitis cases in measles, mumps and german measles vaccinated children
has been shown to be caused by p 1045
A. Herpes simplex virus (0.25)
B. Enteroviruses (0.25)
C. Cytomegalovirus (0.25)
D. All of the above (0.25)

87. A 1 ½ year old male infant sought consultation for fever of 2 days with irritability/drooling of the saliva
and decrease formula/food intake. Oropharyngeal findings showed vesicles and ulcers on the uvula,
soft palate, anterior tonsillar pillars and posterior pharyngeal wall. The most likely diagnosis would be
p 1045
A. Aphthous ulcers (0.5)
B. Herpetic gingivostomatitis
C. Hand-foot and mouth disease
D. Herpangina (0.5)

88. Genital herpes virus infection occurs in p 1053


A. any age group especially school age group
B. children via autoinoculation and sexual abuse (0.5)
C. primary and recurrent disease and can be differentiated clinically
D. a neonate in 90% within the first week of life (0.5)

89. The management of an infant born to a pregnant woman with active genital HSV infection will consist
of p 1056
A. Vaginal delivery for recurrent infection (0.33)
B. Caesarean section within 4-hour of rupture of bag of water (0.33)
C. Infant should be treated immediately with acyclovir without need to perform
cultures (0.33)
D. If sign/s of HSV infection is/are present on the mother, a serologic
determination of her immune status should be performed

90.A 25 year old pregnant woman (+) for Hep Bs Ag delivered a term, AGA male newborn with an Apgar
score of 9 (-1 for color) and 10 at 1 and 5 minutes. What measure will you undertake to prevent
perinatal infection? P 1329
A. Hepatitis B vaccination at 6 weeks, 10 weeks and 14 weeks of age
B. Hepatitis B vaccination at 2 months, 4 months and 6 months of age
C. Hepatitis B vaccination within 12 hours of life plus hyperimmune
globulin against Hepatitis B (1.0)
D. Hepatitis B vaccination before 1 – 2 month of age

91. Neonatal infants of diabetic mothers are at high risk for hypoglycemia because:
A. Hyperinsulinism exists p 613
B. There is impaired gluconeogenesis in response to hypoglycemia
C. Increased metabolic needs disproportionate to substrate stores and calories supplied.
D. All of the above (1.0)

92. Intractable neonatal hypoglycemia in infants with macroglossia, large size, visceromegaly, mild
microcephaly, omphalocoele, facial nevus flammeus, a characteristic earlobe creases are seen in
infants with: p 616
A. Turner syndrome
B. Klinefelter syndrome
C. Down syndrome
D. Beckwith-Wiedemann Syndrome (1.0)

93. Infections are a frequent and important cause of morbidity and mortality in the neonatal period
because:
A. Infectious agents can be transmitted from mother to the fetus or newborn infant by
diverse modes. (0.5)
B. Newborn infants are more capable of responding to infection because of less immunologic
deficiencies
C. Diagnosis and management of neonatal infections are not often complicated by coexisting
conditions
D. Maternal infections that is the source of transplacental fetal infection is often detected in
early pregnancy. (0.5)

94. This is characteristic of a neonate with early onset of sepsis p 628


A. Poor suck, hypothermia, hypoglycemia manifesting at 10th day of life.

44
B. Increased incidence of seizures and bulging fontanelles
C. Multisystem involvement and higher incidence among preterm neonates 1 (1.0)
D. Uncommon maternal obstetric complications

95. Vitamin K deficiency is characterized by p 190


A. low platelet count (0.25)
B. normal protime, abnormal PTT (0.25)
C. normal PTT and abnormal PT (0.25)
D. abnormal PT and PTT (0.25)

96. A prolonged PTT, normal PT and platelet count should make one suspicious of
A. Acquired Prothrombin Complex Deficiency p 1657-1658

B. Disseminated Intravascular Coagulopathy (0.5)


C. Hemophilia (0.5)
D. Idiopathic Thrombocytopenic Purpura

97. This is not usually administered in chronic Idiopathic thrombocytopenic purpura


A. Vincristine sulfate (0.5) p 1670 -1671
B. Danazol
C. Steroids
D. Anthracycline (0.5)

98. Treatment of choice for severe aplastic anemia is


A. Antithymocyte globulin p 1642 - 1644
B. Steroids (0.5)
C. Bone Marrow Transplant (0.5)
D. Androgens

99. Severe joint pains, weight loss, organomegalies, anemia are features of
A. Acute Lymphocytic Leukemia (0.5)
B. Aplastic anemia (0.5)
C. Fanconi’s anemia
D. von Willebrand’s disease

100. Severe hemophilia will present as: p 1657-1658


A. Hemarthroses (0.5)
B. Petecchial rashes
C. Epistaxis (0.5)
D. Ecchymoses
---END---

DEPARTMENT OF PEDIATRICS

E. CARDIO-ENDO

D 1. In diabetes insipidus, the patient’s urine is of:


A. high specific gravity, less volume C. low specific gravity, less volume
B. high specific gravity, large volume D. low specific gravity, large volume
D 2. In endocrine disease:
A. an end-organ defect responds to exagenous hormone
B. all lesions present at birth
C. all features of disease are not reversible
D. hormone replacement is one of its mode of management
D 3. The diastolic duration of the cardiac cycle is longer than the systolic time because:
A. contraction of the heart is time-related
B. the parasympathetic system of the heart is dominant
C. the heart needs short and vigorous contraction to increase the stroke volume
D. the heart needs to relax to increase the LV end-diastolic volume
D 4. Crying, the first independent respiration of the Neonate produces:
A. Immediate closure of all intra and extracardiac shunts
B. Reversal of left-to-right shunts
C. Systemic vasoconstriction
D. Increased arterial oxygen saturation
B 5. The pressure change responsible for the foramen ovale flap closure is due to:
A. increased RA volume C. increased RA pressure
B. increased pulmonary venous return D. increased RV pressure

45
C 6. The cyanosis in Tetralogy of Fallot:
A. always evident at birth
B. is due to intrapulmonary right-to-left shunting
C. is due to the overriding of the aorta and pulmonic stenosis
D. is primarily due to the VSD
C 7. True of Rheumatic Fever
A. Preceeded by any streptococcal infection
B. Usually present with high grade fever
C. May present with migratory arthritis involving large joint leaving no sequelae
D. May present with migratory polyarthritis leaving a joint deformity

F. BACTERIAL INFECTION

B 8. One of the following statements about the treatment of pertussis is true:


A. Exposure to the sea is conventional therapy
B. Ampicillin do not shorten the duration but decreases the communicability
C. The organism is eradicated in the first twenty four hours after intake of antibiotic
D. The current recommended therapy is 7 days
C 9. This organism is motile gram positive anaerobic rods that forms terminal spores
resembling drumsticks. The most common disease presentation is that of:
A. Pharyngitis
B. Tetanus
C. Pseudomembrane formation
D. Pleural Effusion
B 10. A 9-year-old kid attended a kiddie party. He was noted to eat salad, noodles and chicken.
After 4-hours, he was noted to have severe vomiting and diarrhea. A number of people who
attended the party suffered the same fats. The most probable diagnosis is:
A. Streptococcus B. Staphylococcus C. MycoplasmaD. Pneumococcus
-2-
A 11. A 5-year-old child was noted to have spiking fever associated with conjunctivitis. He also
complained of calf pain and develop jaundice in five days time after onset of symptoms. I would
be highly considering:
A. LeptospirosisB. Hepatitis C. Measles D. Chlamydial infection
A 12. Bleeding into the adrenals with septicemia of meningococcemia is seen in:
A. Waterhouse-Friderichsen Syndrome C. Job-Chedeiki Sydnrome
B. Wiskott-Aldrich Syndrome D. Nephrotic Syndrome
B 13. A child had an accident sustaining wounds from a rusty iron bar. After 14 days, he
developed trismus, generalized stiffness and spasms. He was however oriented to time, place
and person. The most probable disease condition affecting the patient is:
A. Rabies C. seizure disorder secondary to CNS infection
B. Tetanus D. DPT shot

G. RESPIRATORY

A 14. Refers to the volume of normal breathing:


A. Tidal volume C. Inspiratory reserve volume D. Expiratory reserve volume
B. Vital capacity E. Residual volume
C 15. A condition characterized by progressive granulation of the small airways:
A. bronchial asthma C. bronchiolitis obliterans D. bronchiectasis
B. acute bronchitis E. Acute bronchiolitis
A 16. The following is/are the characteristics of hydrocarbon aspiration pneumonia:
A. hydrocarbon has a low viscosity and a high volatility properties
B. asymptomatic patient can be send home immediately from the ER
C. routine antibiotics should be given to prevent secondary infection
D. All
C 17. A condition resulting from transudation of fluid from the pulmonary capillary into the
alveolar spaces and bronchioles:
A. Atelectasis C. Pulmonary edema D. Pulmonary infarction
B. Pulmonary Effusion E. Emphysema
C 18. A condition characterized by congenital flabbiness of the epiglottis and subglottic
aperture:
A. Tracheoesophageal fistula
B. Laryngeal webs
C. Laryngomalacia
D. GER
E. Laryngeal Atresia
C 19. The following condition(s) should be considered in a child with unilateral foul smelling nasal

46
discharge:
A. Acute rhinitis
B. Allergic rhinitis
C. Nasal foreign body obstruction
D. Nasal polyps
E. Sinusitis
A 20. The most common cause of obstructive sleep apnea in children (OSA)
A. Tonsillar hypertrophy
B. Obesity
C. Hypotonia
D. Acute Tonsillitis
E. Retropharyngeal abscess

H. VIRAL INFECTION

B 21. Clinical features of a classic congenital rubella syndrome, EXCEPT:


A. mental retardation
B. Hydrocephalus
C. Cataracts
D. Congenital Heart Anomalies
C 22. Lymphadenopathy, may be generalized most commonly suboccipital, post auricular and post
cervical nodes:
A. Roseola Infantum
B. Rubeola
C. Rubella
D. Varicella
B 23. In passive immunization for rabies, there is a high incidence of serum sickness:
A. Human rabies serum globulin
B. Animal rabies serum
C. Duck Embryo Vaccine
D. Human Diploid Cell Vaccine
A 24. Suggestive of Dengue Infection, EXCEPT:
A. Hypertension
B. Hyperpyrexia
C. Hepatomegaly
D. Hemorrhagic Manifestation
C 25. Recommended fluid therapy for DSS:
A. platelet concentrate
B. D50.3 NaCl
C. Plasma expander
D. FWB

I. HEMATOLOGY

A 1-week old male referred to you. Patient was born preterm, developed RDS (Respiratory
Distress Syndrome) hence was intubated. At present patient has prolonged profuse bleeding on all
punctures sites:

B 26. What is the most likely diagnosis:


A. Hemorrhagic Disease of the Newborn
B. Disseminated Intravascular Coagulation
C. Idiopathic Thrombocytopenic Purpura
D. Hemophilia
E. Hemolytic Anemia
B 27. What is/are the most likely pathophysiologic event behind the patient’s condition?
A. there is no carboxylation of specific clotting factors
B. there is tissue necrosis due to widespread fibrin deposition
C. there is transient deficiency of Vitamin K dependent factors
D. there is formation of antibodies of specific clotting factors
D 28. If needed, what is/are the blood component which the patient urgently requires:
A. Packed red blood cell
B. Fresh frozen plasma
C. Platelet concentrate
D. Fresh whole blood
E. Granulocyte concentrate
A 10-year-old MALE admitted due to pallor of 1-month duration. History reveals that his father
and his older brother had an unrecalled/abdominal operation when they were on school age (8-10
years old) PPE: ABW - 20 kg.; (+) splenomegaly.

47
A 29. The following condition(s) should be included in your clinical impression(s) of the case:
A. Hereditary Spherocytosis
B. Thallasemia
C. ITP
D. Aplastic Anemia
E. Congenital Vitamin K deficiency
A 30. The following is/are true of Thallasemia:
A. There exist heterogenous clinical conditions of the illness
B. Opportunistic infections are common complications
C. Transmission of illness is autosomal dominant
D. It may result from idiosyncratic reaction to drugs
E. All of the above

J. NEUROLOGY

D 31. A 4-year-old child with Tuberculous Meningitis 2nd stage will most likely develop one of the
following complications:
A. deafness
B. ventriculitis
C. blindness
D. hydrocephalus
A 32. A 2-week-old infant with bacterial meningitis usually would exhibit:
A. lethargy
B. opisthotonus
C. headache
D. nuchal rigidity
B 33. Partial seizures may have one of the following features:
A. are seen only during the infancy period
B. may or may NOT be associated with loss of consciousness
C. are short duration most of the time
D. not seen in the neonatal period
C 34. A 4-year-old male child developed easy fatigability becoming more prominent in the late
afternoon. Physical examination shows bilateral ptosis. What is the most likely diagnosis?
A. Spinal Muscular Atrophy
B. Becker’s Dystrophy
C. Myasthenia Gravis
D. Polymyositis
B 35. A 6-year-old female child develops diabetes insipidus with severe headache and vomiting.
She most likely has a:
A. Supratentorial tumor
B. Suprasellar tumor
C. Infratentorial tumor
D. Midline tumor
B 36. A 10-year-old child with Duchenne’s Muscular Dystrophy is most likely:
A. able to walk with a stable gait
B. Wheelchair bound
C. Bedridden
D. Able to run
A 37. Joshua is a 9-year old male child who developed moaning sound with tonic clonic
contractions of extremities while has was asleep. What is your primary consideration?
A. Rolandic Epilepsy
B. Complex Partial Seizures
C. Absence
D. Akinetic Seizures

K. NEPHROLOGY

A 38. Among children, it is the most common histologic finding in patients with Nephrotic
Syndrome:
A. minimal change nephrotic syndrome
B. membranous nephropathy
C. membranoproliferative nephrotic syndrome
D. focal segmental glomerulosclerosis
B 39. One of the following statements about hypertension is true:
A. The most common cause of hypertension in children is essential
B. The most common endocrinologic cause of hypertension is pheocromocytoma
C. The appropriate cuff for age covers one third of the upper arm

48
D. The arm while taking hypertension should be at the level of the scapula
D 40. The most common renal consult in the world is:
A. Acute glomerulonephritis
B. Henoch Schoenlein Purpura
C. IgA Nephropathy
D. UTI
B 41. The following statement is true of Idiopathic Hypercalciuria:
A. It may present as recurrent proteinuria
B. Oral thiazides can normalize urinary calcium excretion
C. Patients have elevated serum calcium
D. It is familial
B 42. Persistently low C3 with hard to control hypertension is seen among patients with:
A. Acute glomerulonephritis
B. Membranoproliferative disease
C. Systemic Lupus Erythematosus
D. Bacterial Endocarditis
B 43. In Acute Renal Failure, one of the following is usually observed:
A. Small for age kidneys
B. Appropriate Bone Aging
C. Enlarged Heart
D. Stunted Growth

L. FLUIDS/ELECTROLYTES

B 44. In mild Hypotonic Dehydration, the parenteral hydrating fluid of choice is the one that
should contain:
A. 100 meq/L of sodium
B. 75 meq/L of sodium
C. 50 meq/l of sodium
D. 25 meq/L of sodium
C 45. In Hypertonic dehydration 40-50 meq/L of potassium is provided in the IV fluids not only
to replace potassium losses but also:
A. to hasten entry of water into the cells
B. to prevent Ileus
C. to correct existing metabolic acidosis
D. to prevent hypoglycemia
C 46. The following conditions will give rise to a state of Hyponatremia, EXCEPT:
A. SIADH
B. Acute Renal Failure
C. Congenital Adrenal Hyperplasia
D. Cystic Fibrosis of the Pancreas
B 47. Under normal condition, the daily water expenditure is about 50% is lost in the urine
35% is lost in the skin sweat and:
A. 15% is lost in the lungs
B. 10% is lost in the lungs
C. 5% is lost in the lungs
D. None of the above
C 48. A 10 kg. patients came in shock with severe Hypotonic Dehydration. The initial hydrating
fluid you should give is:
A. D50.45 SS 500 cc
B. Normosol-M 500 cc
C. D5LR 500cc
D. Plasma 500 cc
A 49. The following IV fluids do not contain potassium, EXCEPT; which of the following IV
fluids contain potassium:
A. D5 Ionosol-MB
B. Ringer’s Lactate
C. Normal Saline Solution
D. D50.45% NaCl Solution
D 50. Infants has a higher metabolic rate, about twice the adult and his daily water turnover is
about:
A. 15% of his total body water
B. 20% of his total body water
C. 20% of his body weight
D. 20% of his body surface are

M. NEWBORN

49
A 51. Meconium is expelled:
A. within 24-hours after birth
B. 2 days after birth
C. one week after birth
D. one hour after birth
B 52. A 6-10% weight loss is observed in the newborn infant on:
A. the first day of life
B. one week after birth
C. 2nd week of life
D. 3rd week of life
A 53. The most important maneuver to assess muscle tone is:
A. traction maneuver C. horizontal suspension
B. parachute maneuver D. vertical suspension
B 54. Respiratory Distress Syndrome is most likely to occur in one of the following:
A. adequate for gestational age babies
B. Preterm babies
C. Postmature babies
D. Babies of diabetic mothers
C 55. One of the following is true of Caput Succedaneum:
A. Does not cross suture lines
B. Associated with fracture
C. Transient event which disappears in a few days of weeks
D. Associated with increased intracranial pressure
A 56. One of the following events happen after birth:
A. Placental circulation ceases when cord is clamped
B. Respiration occurs
C. Infants expand lungs
D. Foramen ovale closes
C 57. A 36-weeks old infant assumes one of the following:
A. Hypertonic posture
B. Popliteal angle of 90 degrees
C. Has elbows slightly passing the midline
D. Fisted hands
A 58. It is universally accepted that Hyaline Membrane Disease of the newborn:
A. Is also known as Respiratory Distress Syndrome
B. Is a leading cause of death among term newborn
C. Seen only in premature babies
D. Does not occur in infants of diabetic mothers
A 59. A preterm is a live newborn delivered:
A. before 37 weeks gestation
B. after 37 weeks gestation
C. before 16 weeks gestation
D. after 41 weeks gestation

PREVENTIVE PEDIATRICS

C 60. A solution of antibodies derived from the serum of animals immunized with specific antigen
is:
A. vaccine
B. toxoid
C. antitoxin
D. immuneglobulin
A 61. Which of the following is a contraindication to vaccine?
A. Moderate illness with fever
B. Breastfeeding
C. Diarrhea
D. Prematurity
B 62. Aluminum hydroxide enhances immunogenicity. What is it?
A. suspending fluid
B. adjuvant
C. preservative
D. stabilizer
D 63. If the buttocks are to be used for vaccination, use only the:
A. central region C. upper inner quadrant
B. lower outer quadrant D. upper outer quadrant

A 64. Arthritis after MMR vaccination is due to which component?


A. Rubella

50
B. Measles
C. Mumps
D. Measles and Rubella
A 65. One of the following is a usual occurrence after a BCG response:
A. appearance of wheal
B. induration after 2-3 days
C. pustule after 5 – 7 days
D. scar after 2 – 3 weeks
B 66. If not administered simultaneously, a 4 week minimum interval is needed for these
vaccines:
A. DPT and Hepatitis B
B. MMR and Varicella
C. Hepatitis B and MMR
D. OPV and MMR

N. PEDIATRICS DIAGNOSIS/GENETICS

A 67. Part of the body that should be examined first when the child is resting and cooperative:
A. Lungs
B. Eyes
C. Oropharynx
D. Heart
D 68. The cardiac rate of a child is better counted with:
A. palpation of the wrist
B. palpation of the apex
C. visual chestwall inspection
D. Auscultation
A 69. The chromosomal abnormality of Edward’s Syndrome is due to:
A. Addition of one chromosome
B. Deletion of Chromosome
C. Breakage of Chromosome
D. Abnormality of sex chromosome
A 70. In taking a patient history of “present illness”, informations should begin with:
A. nature and date of onset
B. medication’s patient taken
C. Immunization patient’s received
D. Family living condition
C 71. Composition of milk formula, amount and interval with child’s food intake informations
should be listed in:
A. maternal history
B. birth history
C. feeding history
D. Growth and Development History
C 72. This diagnosis should be considered in a child with prolonged unilateral, foul smelling nasal
discharge:
A. Allergic Rhinitis
B. Acute Nasopharyngitis
C. Foreign body nasal obstruction
D. Nasal Polyps
-9-
O. NUTRITION

D 73. True of minerals, EXCEPT:


A. Sodium is the major regulator of plasma osmolality
B. Potassium is the principal intracellular cation
C. Magnesium is the principal cation of soft tissue
D. Calcium retained in the body 50%
E. Iron intake 90% excreted in the stools
D 74. Which vitamin is incorrectly matched with its advisable daily intake for the infant?
A. Vitamin A 1800 IU
B. Vitamin D 400 IU
C. Vitamin C 75 mg
D. Niacin 0.6 mg
E. Thiamine 0.5 mg
B 75. Tryptophan load test is a screening for deficiency of:
A. Thiamine
B. Pyridoxine
C. Folacin

51
D. Cobalamine
E. Niacin
B 76. Protein content of this type of milk is much higher than breast milk about 21% vs 7-16%:
A. evaporated milk
B. whole cow’s milk
C. condensed milk
D. skimmed milk
E. filled milk
B 77. Amount of nitrogen accumulated compared with nitrogen absorbed which indicates
effectiveness of utilization:
A. Protein efficiency ratio
B. Biologic value of protein
C. Net protein utilization
D. Protein daily requirement
E. Protein adequacy
B 78. Amino acids in excess of needs for protein synthesis are handled in the following ways,
EXCEPT:
A. excreted as ammonia
B. stored as amino acids in the liver
C. synthesized into energy yielding compounds
D. may be synthesized into fats
E. excreted as urea

P. GROWTH/DEVELOPMENT

C 79. In Tanner’s SMR, when the areola and papilla from secondary mound, this is:
A. SMR-2
B. SMR-3
C. SMR-4
D. SMR-5
C 80. Resolution of the oedipal complex should take place by the:
A. Anal stage
B. Phallic Stage
C. Latency Stage
D. Adolescent Stage

A 81. Aspect of behavior most closely related to intelligence:


A. Adaptive
B. Language
C. Fine motor
D. Personal and social
B 82. Abdominal circumference is measured at the level of;
A. xiphoid bone
B. the plane of the umbilicus
C. 2 cm below the xiphoid level
D. 2 cm below the umbilicus
C 83. A seven year old height:
A. 105 cm
B. 110 cm
C. 115 cm
D. 120 cm

B 84. True of sensorimotor stage of Piaget’s Theory, EXCEPT:


A. object permanence
B. egocentricism
C. causality
D. spatial relationship
C 85. A five year old is able to do the following, EXCEPT:
A. copies square
B. count to ten
C. adds and subtracts
D. uses knife
C 86. The peak height velocity for boys occur at Tanners:
A. SMR – 2
B. SMR – 3
C. SMR – 4
D. SMR – 5

52
Q. GIT

D 87. An important factor in many cases of Necrotizing Enterocolitis is:


A. Maternal malnutrition
B. Genetics
C. Environment
D. Neonatal Anoxia follows delivery
D 88. The so called “common shaped” ileum demonstrated on X-ray is a significant finding of:
A. ulcerative colitis
B. necrotizing enterocolitis
C. Hirschsprung’s Disease
D. None of the above
A 89. A condition characterized as having a tender sausages-shaped mass felt in the region of
ascending or transverse colon is seen in:
A. Intussusception
B. Volvulus
C. Meckel’s Diverticulum
D. None of the above
B 90. The cardinal signs and symptoms of intestinal obstruction include all best one:
A. Failure to pass flatus C. Vomiting
B. Failure to pass meconium D. Regurgitation
B 91. The most common virus causing diarrhea in children is:
A. Reovirus B. Rotavirus C. Adenovirus D. Enterovirus
A 92. All but one are characteristic features of Hepatitis B:
A. It is a single stranded RNA virus
B. All age groups affected
C. Fever less common
D. Can infect the fetus or newborn
C 93. The presence of a hard oblong mass along the right rectus muscle is a typical physical
finding seen in:
A. Intussusception
B. Peptic Ulcer
C. Congenital Hypertrophic Pyloric Stenosis
D. None of the above
A 94. True of Tracheoesophageal Atresia/Fistula type B, EXCEPT:
A. Abdominal distention
B. Drooling of saliva
C. Chocking and coughing
D. (+) history of maternal polyhydramnios
D 95. The most common type of Hypoplasia of the mandible is:
A. Vincent’s Angina C. Trench Mouth
B. Pyorrhea D. Pierre-Robin Syndrome

R. ALLERGY/IMMUNOLOGY

B 96. True of Anaphylaxis:


A. bronchospasm specific for illness
B. Antigens have variable routes of entry
C. Common in pediatric age group
D. None of the Above
A 97. True of peripheral eosinophilia:
A. present in atopy
B. specific for atopy
C. absence rules out atopy
D. None of the above
C 98. The following will differentiate urticaria from Allergic Contact Dermatitis:
A. pruritus and lichenification
B. favorable response to H-1 receptor blocker
C. typical morphology of distribution
D. None of the above
C 99. True of Atopic Dermatitis:
A. Characterized by typical distribution of rash according to age group
B. Characterized by different ages of onset
C. Characterized by typical morphology of distribution
D. None of the above
B 100.True of Urticaria:
A. Characterized by typical morphology of distribution
B. Characterized by pruritus

53
C. Characterized by typical distribution of rash according to age group
D. None of the above

E. E N D

1. Which of the following findings is NOT consistent with physical abuse?


A. Implement marks on the torso
B. Bruises of varying ages
C. Symmetric bruises
D. Bruises on the shins, knees and elbows
(Answer: D / Reference: A p 1125 / MPL: 0.6)

2. Which of the following is the best way to manage dermal exposure to a potentially toxic substance?
A. Wash the exposed area with copious amounts of neutralizing solution
B. Wash the exposed are with lots of soap and water
C. Wash the exposed area with copious amounts of lukewarm water
D. Give oral antidote
(Answer: C / Reference: A p 2364 / MPL: 0.6)

3. A 3 year old female was brought to your house because of vomiting, abdominal pain, and nausea.
While examining her, you noted that she had a garlic smell emanating from her mouth. What will you do?
A. Induce emesis
B. Give 6-8 egg whites
C. Call an ambulance and bring her to the hospital immediately
D. Give activated charcoal
(Answer: C / Reference: A p 1426 / MPL: 0.33)

4. Which of the following statements is NOT TRUE regarding nephrotic Syndrome?


A. elevated serum triglycerides
B. may cause iron deficiency anemia
C. normal ionized calcium
D. may have decreased intravascular volume
(Answer: C / Reference: A p 1755 / MPL: 0.25)

5. Which of the following is NOT a typical course for acute post-streptococcal glomerulonephritis?
A. Improvement of the nephritic manifestations can be observed after 1 week of onset
B. Hypocomplementenemia resolves after 6 months
C. Gross hematuria resolves after 2 months
D. Microscopic hematuria may persist for a year
(Answer: B / Reference: A p 1740 / MPL: 0.25)

6. Which of the following is NOT TRUE regarding the Hemolytic Uremic Syndrome?
A. Associated with thrombocytopenia
B. Associated with hematuria
C. Responds to steroid therapy
D. Can be recurrent
(Answer: C / Reference: B p 1746 - 1747 / MPL: 0.33)

7. Which substance is being tested in the Philippine Newborn Screening for Congenital Adrenal
Hyperplasia?
A. Progesterone
B. 17 OH pregnenolone
C. !7 OH progesterone
D. 11 B hydroxylase
(Answer: C / Reference: B 1912 / MPL: 0.6)

8. Which of the following syndromes is associated with elevated growth hormone levels?
A. Sotos syndrome
B. Laron syndrome
C. Klinefelter’s syndrome
D. Marfan’s syndrome
(Answer: B / Reference: B p. 1848 / MPL: 0.25)

9. Which of the following is the drug of choice for complex partial seizures?
A. Carbamazepine
B. Phenytoin
C. Phenobarbital
D. Diazepam

54
(Answer: A / Reference: B p. 2003 / MPL: 0.9)

10. A 700 gm infant presents with abdominal distention, excessive gastric residuals and bloody stools.
Physical examination reveals lethargy, poor perfusion and recurrent apnea. Abdominal examination
reveals tenderness, guarding and erythema. You suspect necrotizing enterocolitis (NEC). Of the following,
the MOST accurate statement about NEC is that:
A. age at onset is inversely related to gestational age at birth
B. it is more common among critically ill than convalescing neonates
C. pneumatosis intestinalis is the earliest radiographic sign
D. strictures that occur as a late complication appear most frequently in the ileum
(Answer: A / Reference: B pp. 590 - 591 / MPL: 0.6)

11. A one month old infant was noted to have a small fontanel. Presence of persistently small fontanel is
seen in all of the following conditions, EXCEPT:
A. cranio synostosis
B. congenital hyperthyroidism
C. congenital rubella syndrome
D. wormian bones
(Answer: C / Reference: B p. 525 / MPL: 0.33)

12. An infant is born full term to a woman who had an uncomplicated pregnancy. Immediately after
delivery, the infant has severe respiratory failure. Breath sounds are diminished bilaterally. The abdomen
is flat. The CXR shows a multicystic mass in the left chest with a shift of the mediastinum to the right.
What is the most likely diagnosis?
A. Congenital lobar emphysema
B. Diaphragmatic hernia
C. Respiratory distress syndrome
D. Persistence of fetal circulation
(Answer: B / Reference: B pp. 1353 - 1355 / MPL: 0.6)

13. A mother is first seen at 28 weeks gestation with severe preeclampsia. A decision is made to deliver
the infant. At birth the infant is placed in 100% oxygen. The first arterial blood gas reveals that the infant is
hypoxic and has elevated PCO2. Which of the following immediate diagnostic and therapeutic plans is
considered the least priority?
A. chest radiograph
B. exogenous surfactant
C. provide mechanical ventilation
D. provide volume expansion
(Answer: D / Reference: B pp. 570 - 572 / MPL: 0.6)

14. An infant born at 30 weeks gestation begins to experience apnea on the second day of life. Which of
the following is the least appropriate initial management option for this infant?
A. therapy with theophylline
B. evaluation of evidence of hypoxia, infection or intracranial hemorrhage
C. CBC, arterial blood gas studies, plasma glucose and electrolyte measurement
D. Check that the environmental temperature is in the neutral thermal zone
(Answer: A / Reference: B pp. 573 - 574 / MPL: 0.33)

15. An infant is born 28 weeks and weighs 1028 grams. The infant is started on enteric feedings at 3 days
of age. Three days later, he is not tolerating his feedings. Which of the following signs and symptoms
exhibited by the infant is the least specific for neonatal NEC?
A. bile-stained gastric fluid
B. pneumatosis intestinalis
C. abdominal distention
D. jaundice
(Answer: D / Reference: B pp. 590 - 591 / MPL: 0.6)

16. An infant was born at 26 weeks gestation. The infant is now 8 weeks old and has broncho-pulmonary
dysplasia and retinopathy of prematurity (ROP). Which of the following statements is NOT TRUE of the
pathogenesis and treatment of ROP?
A. its development is related to retinal vessel immaturity and hyperoxia.

55
B. Cryotherapy is the treatment of choice for all stages of disease.
C. The retinopathy resolves spontaneously in most infants.
D. Myopia is a common sequelae.
(Answer: B / Reference: B pp. 2113 - 2114 / MPL: 0.33)

17. An infant in the delivery room is noted to have respiratory movements, but no air is entering the lungs
with the mouth closed. Which of the following is the MOST likely diagnosis?
A. narcosis
B. choanal atresia
C. pulmonary hypoplasia
D. congenital heart disease
(Answer: B / Reference: B 1386 - 1387 / MPL: 0.9)

18. Jaundice is most likely physiologic in a term infant in which one of the following situations?
A. jaundice at 12 hrs of age
B. serum bilirubin level increasing less than 5mg/dL/day in the first 2 to 4 days
C. jaundice at 12 days of age
D. direct serum bilirubin greater than 1mg/dL
(Answer: B / Reference: B 594 - 595 / MPL: 0.6)

C19. A 900 gm infant of 27 weeks gestation developed respiratory distress syndrome and required
endotracheal intubation on the 1st day of life. At 36hours of age, the infant developed hypotension,
bradycardia, cyanosis and tense anterior fontanel. The most appropriate diagnostic test is:
A. EEG
B. Echocardiography
C. Ultrasound of the head
D. Serum coagulation profile
(Answer: C / Reference: B pp. 562 - 564 / MPL: 0.6)

For Questions 20 to 21: A 3-day-old male neonate was noted to have pallor and jaundice. The baby was
otherwise active, with good suck, good urine output. Stools were greenish black in color. There was no
hepatosplenomegaly. The mother’s blood type is “O” Rh+ and the baby’s blood type is also “O” Rh +.
Coomb’s test was negative. Hemoglobin was 11 g/dL and reticulocyte count was 7%. The smear showed
normocytic normochromic red cells with macrocytes noted.

20. What is the most likely diagnosis?


A. Alpha Thalassemia major
B. Breastfeeding jaundice
C. Glucose 6 phosphate dehydrogenase deficiency
D. Physiologic jaundice
(Answer: C / Reference: B 1636 - 1638 / MPL: 0.33)

21. Which of the following test would you order next?


A. Glucose 6 phosphate dehydrogenase assay
B. Hemoglobin electrophoresis
C. Hold breastfeeding and give milk formula
D. Observation and reassure the mother
(Answer: A / Reference: B pp 1636 - 1638 / MPL: 0.33)

22. A 6 hour old term infant delivered spontaneous cephalic was noted to have a swelling over the parietal
area which does not cross the suture lines. The swelling pulsates and becomes tense on crying. Which of
the following is the most likely diagnosis?
A. Caput succedaneum
B. Cephalhematoma
C. Cranial meningocele
D. Molding
(Answer: C / Reference: B 1985 - 1986 / MPL: 0.9)

B23. Which of the following is the recommended vaccination schedule for neonates born to HBsAg
positive mothers?
A. Hepatitis B immunoglobulin vaccine within the 12 hours after birth, Hepatitis B vaccine at 1
month, 3 months and 6 months old
B. Hepatitis B Immunoglobulin vaccine at birth and Hepatitis B vaccine at birth, 1 month and 6
months old
C. Hepatitis B vaccine at birth, one and six months old

56
D. None since mother is not infectious
(Answer: B / Reference: B pp 1328 - 1929 / MPL: 0.6)

For questions No 24-27. A 2-year-old male infant was noted to have intermittent episodes of cyanosis. He
was noted to assume a squatting position while playing. The mother claimed he was not a “blue baby” at
birth and he has no recurrent episodes of cough.

24. A chest x-ray of the patient was done. Which of the following finding would be seen in this patient?
A. Boot shaped heart
B. Normal heart but with increased pulmonary vascularity
C. Heart described as egg on the side
D. Left ventricular hypertrophy
(Answer: A / Reference: B pp 1524 - 1528 / MPL: 0.9)

25. Which of the following is true regarding the above patient?


A. Patient is prone to cerebrovascular event secondary to thrombosis
B. Congestive heart failure is a frequent complication
C. The problem will resolve spontaneously at 5 years of age
D. Bounding pulses is a characteristic feature
(Answer: A / Reference: B pp 1524 - 1528 / MPL: 0.6)

26. An electrocardiogram was done on the patient. Which of the following findings would be true in this
patient?
A. Dominant R waves in the right precordial chest leads
B. Left axis deviation
C. Negative T waves in V3R and V1 leads
D. Peaked R waves in lead II
(Answer: A / Reference: B pp 1524 - 1528 / MPL: 0.33)

27. Which of the following drugs is useful in preventing hypoxic spells in the above patient?
A. Aspirin C. Captopril
B. Dopamine D. Propanolol
(Answer: D / Reference: B pp 1524 - 1528 / MPL: 0.6)

Questions 28-29. While at school a 6 year old boy is noted by his teacher to experience 10-20 second
lapses in consciousness, sometimes with clonic movements of the face. His parents have not noticed this
behavior at home. EEG shows 3 per second generalized spike and wave discharges.

28. What is the MOST likely diagnosis of this boy?


A. absence seizures C. attention deficit disorder
B. complex partial seizures D. generalized tonic clonic seizure
(Answer: A / Reference: B pp. 1997 / MPL: 0.6)

29. Which of the following drugs is the MOST appropriate therapy for this boy?
A. carbamazepine C. gabapentin
B. ketogenic diet D. valproic acid
(Answer: D / Reference: B pp 2003 / MPL: 0.6)

Question Nos. 30-32. A 1-month-old infant was admitted because of cough and respiratory distress.
Pertinent PE findings include rales on both lung fields and a loud continuous murmur. The apical impulse
was also noted to be prominent. Pulses were strong. Mother claimed there was no history of cyanosis but
she noted that the baby tires easily during feeding.

30. The chest x-ray of the above patient will most likely show which finding?
A. left to right shunting with increased pulmonary vascularity
B. prominent pulmonary artery with increased pulmonary vascularity
C. normal heart with hyperaeration of both lung fields
D. right ventricular hypertrophy
(Answer: A / Reference: B pp. 1520 - 1512 / MPL: 0.6)

B31. Which of the following statements is true in this patient?


A. Right atrial hypertrophy is a most likely consequence
B. Blood from the aorta is shunted to the pulmonary artery
C. Brain abscess is a frequent complication
D. Pressure in the right ventricle is usually decreased
(Answer: B / Reference: B pp 1510 - 1512 / MPL: 0.6)

32. Which of the following management is the most applicable in this patient?

57
A. Surgical closure before 1 year of age
B. Pharmacologic therapy with indomethacin
C. Control the heart failure and do surgical closure at 7 years old
D. Surgical closure at 2 years old if there’s no spontaneous closure
(Answer: A / Reference: B pp 1510 - 1512 / MPL: 0.33)

33. A previously healthy and developmentally normal 18 month old boy presents with status epilepticus.
There is no past or family history of seizures. The child is stabilized and the seizure is halted. Subsequent
PE reveals a playful child who has a temperature of 39.2C and nasal discharge. Findings on the
remainder of the examination are normal. Which of the following is the most likely cause of the child’s
status epilepticus?
A. bacterial meningitis C. fever
B. cortical dysplasia D. brain tumor
(Answer: C / Reference: B pp. 2007 / MPL: 0.6)

34. Which of the following is TRUE regarding benign febrile seizures?


A. Peak age of onset is 14 to 18 months
B. Associated with high incidence of epilepsy
C. Anticonvulsants is indicated to prevent recurrences
D. Seizures is typically focal lasting for less than 10 minutes
(Answer: A / Reference: B p 1994 / MPL: 0.9)

35. Which of the following is the most appropriate age-independent measure of protein-calorie
malnutrition?
A. Arm span and reach
B. Height for weight ratio
C. Weight for age
D. Weight for height ratio
(Answer: D / Reference: A p 165 / MPL: 0.6)

36. Which of the following is NOT a characteristic of Kwashiorkor?


A. Dermatitis
B. Edema
C. Photosensitive skin
D. Lethargy, apathy
(Answer: C / Reference: A p 168 - 170 / MPL: 0.9)

37. A patient with Albright’s hereditary osteodystrophy (pseudohypoparathyroidism) sees his physician for
an upper respiratory tract infection. This patient has a stocky build with a round face, short stature and
brachydactyly of the fourth and fifth metacarpals. Which of the following is also associated with
pseudohypoparathyroidism?
A. Mental retardation
B. Decreased serum parathormone
C. Increased serum calcium
D. Decreased serum phosphorus
(Answer: A / Reference: B pp 1894 - 1895)

38. A 2-year-old child is brought to the emergency room for suspected physical abuse. The physician
performs a thorough history and physical examination to determine evidence of maltreatment. Which of
the following would be specific evidence of child abuse in this case?
A. A blue, non tender macular lesion in the presacral area
B. Weight less than the 5th percentile
C. Bruised knees
D. A circular burn restricted to the buttocks
(Answer: D / Reference: A p 1125 / MPL: 0.6)

39. Which of the following is true regarding reporting child abuse incidents?
A. Suspected child abuse should be confirmed before reporting
B. Only the attending physician can report a child abuse case
C. Child abuse cases should be reported within 48 hours of learning of the case
D. Failure to report is punishable by a fine of P500.00 and imprisonment for 1 month
(Answer: C / Reference: A p 1119 / MPL: 0.6)

40. Which of the following would indicate Tanner Stage 3 sexual development in females?
A. Acne
B. Darkly pigmented slightly curly pubic hair
C. Fine hair on the upper lip
D. Menstruation

58
(Answer: B / Reference: A p 53 - 54 / MPL: 0.6)

For question nos. 41 to 46. The 18-month-old daughter of a famous sportsman from Baguio was rushed
to the emergency room because of acute onset of fever associated with purpura and lethargy. The
extremities were cold and pulses were faint. There was profuse bleeding coming from both nostrils.
Meningococcemia with DIC was the consideration. The patient is the 12 th case with meningococcemia in
the locality.

41. Which strain of Neisseria meningitidis is mostly likely to be responsible for the disease in the above
patient?
A. Group A C. Group B
B. Group C D Group W
(Answer: C / Reference: A p 455 / MPL: 0.33)

42. Which of the following is NOT TRUE regarding the above etiologic agent?
A. gram negative diplococci C. obligate intracellular anaerobes
B. polysaccharide capsule D. transmitted via respiratory droplets
(Answer: C / Reference: B p 896 / MPL: 0.9)

43. Which of the following laboratory findings is NOT consistent with the above patient?
A. thrombocytopenia C. Increased fibrinogen
B. prolonged PT and PTT D. increased D-dimers
(Answer: C / Reference: B pp 896 - 899 / MPL: 0.6)

44. Which of the following additional signs and symptoms is NOT expected to be seen in this patient?
A. Pallor C. Peripheral cyanosis
B. Hypotension D. Capillary refill time of 2 seconds
(Answer: D / Reference: B pp 896 – 899 / MPL: 0.6)

45. Which of the following should be the first management priority in this patient?
A. Secure the airway C. Vascular access
B. Fluid resuscitation D. Inotropes
(Answer: A / Reference: B p 296 / MPL: 0.6)

46. Which of the following antibiotics is NOT appropriate for the above patient?
A. Cefotaxime C. Ceftriaxone
B. Chloramphenicol D. Cefalexin
(Answer: D / Reference: A p 896 - 899 / MPL: 0.6)

For questions 47 to 48. A 10 year old girt has a history of easy fatigability, joint pains and swelling of the
ankles, high grade fever, systolic murmur at the apex and a positive ASO titer.

47. Which of the following is the correct diagnosis?


A. RHD with mitral stenosis
B. Acute rheumatic fever with arthritis
C. Acute rheumatic fever with carditis and arthritis
D. RHD with cardiac tamponade
(Answer: C / Reference: B pp. 874 - 879 / MPL: 0.6)

48. Which of the following is the most appropriate management?


A. bed rest, diuretics, penicillin
B. oxygen, morphine, NSAIDs
C. bed rest, steroids, surgery
D. Pen G, bed rest, paracetamol
(Answer: D / Reference: A p 874 – 879 / MPL: 0.6)

49. Which of the following is NOT a diagnostic criteria Kawasaki’s disease?


A. Polymorphous exanthem
B. Bilateral conjunctival injection with exudates
C. Spiking fever up to 40 C
D. unilateral cervical lymphadenopathy
(Answer: B / Reference: B pp 824 - 825 / MPL: 0.6)

50. Which of the following is a sign and/or symptom of congestive heart failure in infants?
A. diaphoresis C. feeding difficulties
B. neck vein distention D. Bipedal edema
(Answer: C / Reference: B p/ 1583 / MPL: 0.6)

59
Questions 51 to 52: .A 5-year-old girl presents at the ER with gross hematuria and periorbital edema. Her
BP is 150/95mmHg.The attending physician suspects she has acute postinfectious glomerulonephritis.
Urinalysis reveals too numerous to count RBCs and 2+ protein. To confirm the diagnosis, you measure
serum complement levels: C3 is 57mg/dl (low) and C4 is 24mg/dl(normal).

51. What is the most common cause of postinfectious glomerulonephritis?


A. Staphylococcus aureus C. adenovirus
B. group A beta hemolytic strep D. Streptococcus epidermidis
(Answer: B / Reference: A p 1743 - 1744 / MPL: 0.6)

52. When do you expect the hypertension to resolve?


A. 2 weeks after onset C. 2 months after onset
B. 4 weeks after onset D. 1 year after onset
(Answer: B / Reference: A p 1743 - 3744 / MPL: 0.6)

For question numbers 53 to 54. A 12-year- old boy presents to the emergency department with a 3 day
history of tea colored urine that began 2 days following an upper respiratory tract infection. There is no
history of dysuria and findings on PE are normal. The family history is negative for any kidney disease.
His BP is 115/70mmHg. Urinalysis reveals: sp gravity 1.025; pH 6.0; RBC too numerous to count; 1+
protein; and 0 to 2 WBC. Electrolyte levels are normal.

53. What is the most likely diagnosis?


A. Alport syndrome C. immunoglobulin A nephropathy
B. focal segmental glomerulosclerosis D. postinfectious glomerulonephritis
(Answer: C / Reference: B pp 1737 - 1738 / MPL: 0.3)

54. Which of the following statements is NOT TRUE of the above patient?
A. renal function is normal C. more commonly seen in males
B. C3 is elevated D. bedrest is part of the management
(Answer: B / Reference: A p 1737 - 1738 / MPL: 0.6)

55. Which of the following is true regarding laryngotracheomalacia?


A. Stridor is usually expiratory in timing
B. Usually resolves by 18 months of age
C. Secondary to RSV
D. Supine position will diminish the intensity of stridor
(Answer: B / Reference: A p 653 / MPL: 0.6)

56. Which of the following is NOT TRUE regarding acute epiglottitis?


A. The most common etiologic agent is Hemophilus influenzae
B. Examination of the throat may precipitate cardiorespiratory arrest
C. Associated with hyperextension of the neck and drooling of saliva
D. Steeple sign is seen on lateral neck x-rays
(Answer: D / Reference: A p 1406 / MPL: 0.6)

57. What is the ideal weight of an 18 month old female child?


A. 7 kg
B. 9 kg
C. 11 kg
D. 15 kg
(Answer: C / Reference: B pp. 31 / MPL: 0.6)

58. Which component of DPT vaccine is highly responsible for the adverse effects after vaccination?
A. Diphtheria
B. Pertussis
C. Typhoid
D. Tetanus
(Answer: B / Reference: A p 37 / MPL: 0.9)

59. Which of the following is the correct definition of a fully immunized child according to EPI?
A. At the age of 1 year, the child should receive 1 dose of BCG, 3 doses of DPT/polio/hepa B and 1
dose of measles
B. At the age of 1 year, the child should receive 1 dose of BCG, 3 doses of DPT/polio/hepa B/HiB
and 1 dose of measles
C. At the age of 1 year, the child should receive 1 dose of BCG, 3 doses of DPT/polio/hepa
B/HiB/Pneumococcal and 1 dose of measles
D. At the age of 1 year, the child should receive 1 dose of BCG, 3 doses of DPT/polio/hepa B and A
and 1 dose of measles

60
(Answer: A / Reference: A p 41 / MPL: 0.9)

60. Which of the following is true regarding Rabies infection?


A. The incubation period ranges from 20-180 days
B. Pre-exposure prophylaxis consists of a 5 dose vaccine regimen
C. Rabies immunoglobulin is indicated for all dog bites
D. Associated with 75% mortality rate

61. Which of the following is the most common manifestation of Candidiasis in children?
A. Diaper dermatitis
B. Pneumonia
C. Urinary tract infection
D. Otitis media
(Answer: A / Reference: A p 1031 / MPL: 0.9)

62. When dealing with bacterial infections, which of the following is the gold standard for diagnosis?
A. Serologic testing
B. Latex agglutination test
C. Viral studies
D. Culture and sensitivity
(Answer: D / Reference: B p 835 - 837 / MPL: 0.9)

63. The presence of generalized muscular spasm, trismus, opisthotonus, and laryngospasm are
characteristic of which etiologic agent?
A. Treponema pallidum
B. Clostridium tetani
C. Bordetella pertussis
D. Corynebacterium diphtheriae
(Answer: B / Reference: A p 468 - 469 / MPL: 0.9)

64. Which of the following is the drug of choice for the above etiologic agent (No. 63)?
A. Penicillin G
B. Tetracycline
C. Amoxycillin
D. Erythromycin
(Answer: A / Reference: A p 469; B p 952 / MPL: 0.9)

65. A 7-year-old girl is brought to you for chronic cough, low grade fever and mild weight loss for more
than a month. Which of the following is your primary consideration?
A. Streptococcus pneumoniae infection
B. Pertussis
C. Mycobacterium tuberculosis infection
D. Influenza
(Answer: C / Reference: A p 503 / MPL: 0.9)

66. Which of the following is NOT a common clinical manifestation of Infection with Hemophilius
influenzae?
A. Meningitis
B. Septic arthritis
C. Urinary tract infections
D. Epiglottitis
(Answer: C / Reference: B pp 905 - 907 / MPL: 0.6)

67. Which of the following is characteristic of roseola iinfantum?


A. Macular rash after fever of 3 days duration
B. High grad fever and rashes occurring at the onset of fever
C. Lymphadenopathy, low grade fever
D. Vesiculopustular rashes after defervescence
(Answer: A / Reference: B p. 1070 / MPL: 0.9)

68. Which of the following is NOT part of the management of children with Rubeola?
A. Antistaphylococcal antibiotics for bronchopneumonia
B. Vitamin A supplementation
C. Isolation until 4 days after the appearance of rash
D. Give Measles vaccination on recovery
(Answer: D / Reference: B p 1029 / MPL: 0.9)

69. Which of the following is NOT true regarding mumps?

61
A. Commonly affects the parotid gland
B. Infectious up to 5 days after onset of swelling
C. 2 doses of MMR vaccine confers full protection
D. All males infected with mumps will develop orchitis
(Answer: D / Reference: B p. 1036 / MPL: 0.6)

70. Which of the following is TRUE regarding Poliovirus infection?


A. OPV administration results in fecal viral excretion for a few days
B. OPV can result in vaccine associated paralytic polio
C. Mass vaccination requires the use of IPV
D. IPV is more potent than OPV
(Answer: B / Reference: B p. 1042 / MPL: 0.6)

71. Rubella vaccination is contraindicated in all of the following, EXCEPT?


A. Pregnant women
B. Women of child-bearing age
C. Patients on prednisone
D. Patients given chemotherapy
(Answer: B / Reference: B p 1033 / MPL: 0.9)

72. A 6 month old infant came in for 4 days moderate grade fever, dry hacking cough and coryza. Koplik
spots were seen on examination of the oral cavity. Which part of the oral cavity are the Koplik spots best
appreciated?
A. opposite the lower canines
B. opposite the lower molars
C. palate
D. peritonsillar areas
(Answer: B / Reference: B p 1027 / MPL: 0.9)

73. Lifetime immunity is acquired by recovery from the following viral infection EXCEPT:
A. Mumps C. Rubella
B. Dengue fever D. Varicella
(Answer: B / Reference: B p 1033, 1036, 1061 / MPL: 0.9)

74. A 3-year-old female child who has been receiving chemotherapy for acute lymphocytic leukemia was
exposed to her sibling with chicken pox. Mother claimed she was not given immunization. Which
prophylactic measure should be administered to this child?
A. Varicella vaccine immediately
B. Varicella zoster immunoglobulin immediately
C. Oral acyclovir
D. None, just isolate the child from the sibling
(Answer: B / Reference: A p 1061 - 1062 / MPL: 0.6)

75. Which of the following is NOT true regarding varicella vaccine?


A. it should be given intramuscularly
B. may be given simultaneously with other vaccines
C. Contains a live attenuated strain of varicella virus
D. 2 doses are required for children < 12 years old
(Answer: D / Reference: C p 680 - 681 / MPL: 0.6)

76. Which of the following is considered the infectious period of varicella?


A. 1 day before and 5-7 days after the onset of rash
B. 1 week before and 2 weeks after the rash
C. during the appearance of the rash and 2 weeks after
D. during the prodrome period and 10 days after the disappearance of the rash
(Answer: A / Reference: B p 1057 / MPL: 0.6)

77. Which of the following is not an eye sign of avitaminosis A ?


A. Bitot’s spots
B. Nyctalopia
C. Xerophthalmia
D. Keratomalacia
(Answer: B / Reference: B p. 180 / MPL: 0.9)

78. Which of the following is NOT characteristic of the keratoconjunctivitis caused by Herpes simplex
virus?
A. painful vesicles on the lids
B. enlarged and tender preauricular nodes

62
C. keratomalacia
D. fluorescent dendritic ulcers of the cornea
(Answer: C / Reference: B p 1053 / MPL: 0.6)

For question numbers 79-80. A 2 year old female was brought to the emergency room with high grade
fever, and diffuse rash (onset of 5 days ago), and a swollen right hand. The rashes were noted to be
vesiculopustular over the trunk, face and extremities. Crusting were noted in some of the lesions. The
right hand was noted to swollen, erythematous and tender. CBC showed leukocytosis.

79. Which of the following would be the most appropriate therapeutic measures for the patient?
A. Give oral acyclovir
B. Give intravenous cephalexin
C. Give intravenous acyclovir
D. Paracetamol only
(Answer: B / Reference: B p. 1061 / MPL: 0.9)

C80. Which of the following is the main mode of transmission of the causative agent for the diffuse rashes
of the patient?
A. Skin to lesion contact
B. Oral-fecal route
C. Respiratory route
D. Local inoculation
(Answer: C / Reference: B p 1057 / MPL: 0.9)

81. The most characteristic sign of rubella:


maculopapular rash on the trunk and extremities
Discrete rose spots in the soft palate
Retroauricular, posterior cervical and postoccipital lymphadenopathy
Branny desquamation
(Answer: C / Reference: B p. 1032 / MPL: 0.9)

82.. The cardiac structural defects associated with congenital rubella syndrome:
A. patent ductus arteriosus and ventricular septal defect
B. atrial septal defect and ventricular septal defect
C. ventricular septal defect and pulmonary artery stenosis
D. pulmonary artery stenosis and patent ductus arteriosus
(Answer: D / Reference: B p 1033 / MPL: 0.6)

83. Which of the following conditions does not present with fetal infection?
A. Rubella
B. Rubeola
C.Parvovirus B19 infection
D. Varicella Zoster virus infection
(Answer: B / Reference: B p. 1033, 1050, 1059 / MPL: 0.6)

84. The highest incidence of diarrhea occurs in this age group:


A. 0-5 months C. 12-17 months
B. 6-11 months D. 18-24 months
(Answer: B / Reference: D / MPL: 0.9)

85. The pathogenic mechanism involved in rotavirus diarrhea is:


A. loss of monosaccharidases
B. mucosal adhesion and invasion
C. patchy destruction of the villous epithelium
D. production of toxin which increases secretory function
(Answer: C / Reference: D / MPL: 0.9)

86. The drug of choice for cholera:


A. Cotrimoxazole C. Furazolidone
B. Tetracycline D. Metronidazole
(Answer: B / Reference: D / MPL: 0.9)

87. The first teeth to erupt are usually the:


A. central incisors C. first molars
B. lateral incisors D. second molars
(Answer: A / Reference: A p 60 / MPL: 0.9)

88. Which of the following senses is most developed in newborns?

63
A. auditory C. taste
B. tactile D. visual
(Answer: B / Reference: A p 53 - 54 / MPL: 0.6)

89. Visual acuity of 20/20 is achieved by:


A. 2 yrs old C. 5 yrs old
B. 4 yrs old D. 10 yrs old
(Answer: C / Reference: B p. 2084 / MPL: 0.6)

90. A 3 year old can perform all of the following, EXCEPT:


A. draws man with 3 parts
B. rides a tricycle
C. tells little stories about experiences
D. helps in simple house task
(Answer: A / Reference: A p 112 / MPL: 0.6)

91. A 15 month old is able to do all of the following, EXCEPT:


A. seats self in child’s chair
B. pats pictures
C. attempts to use a spoon
D. imitates a vertical stroke
(Answer: A / Reference: A p 111 / MPL: 0.6)

92. What is the best stimulus to breastmilk secretion in lactating mothers?


A. plenty of fluids and vitamins
B. good rest and high caloric intake
C. eat more shellfish such as clams and tahong
D. frequent sucking and complete emptying of breast
(Answer: D / Reference: A p 142 / MPL: 0.9)
93. Irritability, pruritus, painful extremities with brawny swelling, coarse hair, dry skin, seborrhea and
increased intracranial pressure is seen in:
A. Hypervitaminosis A C. Hypovitaminosis D
B. Hypervitaminosis D D. Hypocalcemia
(Answer: A / Reference: A p 1135 / MPL: 0.6)

94. A 7-year- old presents with the following manifestations: skin and mucous membrane lesions,
conjunctivitis, glossitis, depression and somnolence. What is the most likely deficiency?
A. Vitamin B1 C. Vitamin B12
B. Vitamin B6 D. niacin
(Answer: B / Reference: B p. 183 / MPL: 0.6)

95. Which of the following can be considered as a (+) Mantoux test?


A. A 2 year old boy, no BCG, Mantoux 4mm induration
B. A 4 year old girl, (+) BCG, Mantoux 8mm induration
C. A 2 month old male infant, (+) BCG, Mantoux 3mm induration
D. A 4 year old boy, (+) BCG, Mantoux 12mm induration
(Answer: D / Reference: A p 505 / MPL: 0.33)

96. Which laboratory tests are helpful in establishing the etiology of anemia?
A. hemoglobin, reticulocyte count, bone marrow exam
B. serum iron, CBC, platelet count
C. CBC, serum iron, reticulocyte count
D. CBC, smear review, reticulocyte count, iron studies
(Answer: D / Reference: A p 1065 / MPL: 0.6)

97. Which of the following is NOT a feature of hereditary spherocytosis?


A. >20% spherocytes
B. sex linked recessive
C. splenomegaly
D. cholelithiasis
(Answer: B / Reference: A p 1620 - 1621125 / MPL: 0.6)

98. Which of the following is NOT a feature of aplastic anemia?


A. leucopenia
B. thrombocytopenia
C. hypoferritinemia
D. neutropenia
(Answer: C / Reference: A p 1288 – 1289; MPL: 0.9)

64
99. Which of the following is NOT a favorable prognostic factor of acute lymphoblastic leukemia?
A. 5 years old C. Hemoglobin: 11 gms
B. female D. absence of CNS leukemia
(Answer: C / Reference: A p 1694 - 1696

100. The most important index for protein malnutrition at present is:
A. edema C. negative nitrogen balance
B. wasting D. slow growth and development
Answer: A

65

Das könnte Ihnen auch gefallen